aamc mcat test 8 a

62
1. With respect to bonding and electrical conductivity, respectively, sulfur hexafluoride, SF 6 (g), would be described as: A ) covalent and a nonconductor. The passage states that the vast majority of covalent compounds are comprised exclusively of nonmetallic elements, whereas binary ionic compounds are made up of a metal and a nonmetal. Because neither sulfur nor fluorine is a metallic element, sulfur hexafluoride is a covalent compound. The passage also states that aqueous solutions of covalent compounds do not conduct electricity. Sulfur hexafluoride is covalent and a nonconductor. Thus, A is the best answer. B ) ionic and a nonconductor. C ) covalent and a conductor. D ) ionic and a conductor. 2. Where are nonmetals found in the periodic table? A) Right side The noble gases and halogens are found on the right side of the table. These are nonmetals. Thus, A is the best answer. B ) Left side C ) Top half D ) Bottom half 3. Which of the following pairs of compounds provides an example of ionic and covalent bonding, respectively? A ) HBr(g) and NaCl(s) B ) NaCl(s) and NaI(s) C ) NaI(s) and NaCl(s) D ) NaCl(s) and HBr(g) The passage states that the vast majority of covalent compounds are comprised exclusively of nonmetallic elements, whereas binary ionic compounds are made up of a metal and a nonmetal. Sodium is a metal and chlorine is a nonmetal; consequently, NaCl is an ionic compound. Hydrogen and bromine are both nonmetals, which means HBr is a covalent compound. Thus, D is the best answer. 4. Which of the following compounds has the most ionic character? A ) KBr(s) B ) CsCl(s) The compound with the greatest difference in electronegativities between the metal and nonmetal has the most ionic character. The data in Table 1 show that electronegativities tend to decrease down a group of the periodic table. Cs would have an electronegativity of approximately 0.8 or lower and Cl has an electronegativity of 3.0. The difference between the electronegativities of these two elements is the greatest of the compounds listed. Thus, B is the best answer. C ) NaI(s) D ) RbBr(s) 5. Which of the following statements is consistent with the incorrect conclusion that HCl is an ionic compound? A ) It is a gas at room temperature. B ) A 1 M solution freezes below 0C.

Upload: jennyeunbee

Post on 27-Dec-2015

785 views

Category:

Documents


24 download

DESCRIPTION

MCAT Test 8 answers

TRANSCRIPT

Page 1: AAMC MCAT Test 8 A

1. With respect to bonding and electrical conductivity, respectively, sulfur hexafluoride, SF6(g), would be described as:

A ) covalent and a nonconductor.

The passage states that the vast majority of covalent compounds are comprised exclusively of nonmetallic elements, whereas binary ionic compounds are made up of a metal and a nonmetal. Because neither sulfur nor fluorine is a metallic element, sulfur hexafluoride is a covalent compound. The passage also states that aqueous solutions of covalent compounds do not conduct electricity. Sulfur hexafluoride is covalent and a nonconductor. Thus, A is the best answer.

B ) ionic and a nonconductor.

C ) covalent and a conductor.

D ) ionic and a conductor.

2. Where are nonmetals found in the periodic table?

A) Right side

The noble gases and halogens are found on the right side of the table. These are nonmetals. Thus, A is the best answer.

B ) Left side

C ) Top half

D ) Bottom half

3. Which of the following pairs of compounds provides an example of ionic and covalent bonding, respectively?

A ) HBr(g) and NaCl(s)

B ) NaCl(s) and NaI(s)

C ) NaI(s) and NaCl(s)

D ) NaCl(s) and HBr(g)

The passage states that the vast majority of covalent compounds are comprised exclusively of nonmetallic elements, whereas binary ionic compounds are made up of a metal and a nonmetal. Sodium is a metal and chlorine is a nonmetal; consequently, NaCl is an ionic compound. Hydrogen and bromine are both nonmetals, which means HBr is a covalent compound. Thus, D is the best answer.

4. Which of the following compounds has the most ionic character?

A ) KBr(s)

B ) CsCl(s)

The compound with the greatest difference in electronegativities between the metal and nonmetal has the most ionic character. The data in Table 1 show that electronegativities tend to decrease down a group of the periodic table. Cs would have an electronegativity of approximately 0.8 or lower and Cl has an electronegativity of 3.0. The difference between the electronegativities of these two elements is the greatest of the compounds listed. Thus, B is the best answer.

C ) NaI(s)

D ) RbBr(s)

5. Which of the following statements is consistent with the incorrect conclusion that HCl is an ionic compound?

A ) It is a gas at room temperature.

B ) A 1 M solution freezes below 0°C.

Page 2: AAMC MCAT Test 8 A

C ) A 1 M solution conducts electricity.

The passage states that an aqueous solution of an ionic compound conducts electricity. Thus, C is the best answer.

D ) It is composed of two nonmetals.

6. According to the passage, the magnitude of B3 at the position of a given hydrogen nucleus is determined by the:

A ) chemical environment of the nucleus.

The passage states that B3 is the vector sum of the magnetic fields of the electrons and other nuclei near the H atom, so the chemical composition of the material around the H nucleus being examined will contribute to B3. Only Foil A relates to the material around the H nucleus. Thus, A is the best answer.

B ) mass of the nucleus.

C ) radius of the nucleus.

D ) charge of the nucleus.

7. Consider an H nucleus with µ pointing in a direction 180° from a magnetic field. When the nucleus relaxes, which of the

following most likely will result?

A ) The magnitude of µ will decrease to zero.

B ) The magnitude of the magnetic field will decrease to zero.

C ) The nucleus will emit a photon.

Based on the passage, µ is an intrinsic property of the atomic and subatomic particles, and the magnetic fields are either externally controlled (B1, B2) or intrinsic to the electrons and nuclei (B3). The passage also states that energy is emitted when an H nucleus relaxes, which is compatible only with emitting a photon. Thus, C is the best answer.

D ) The nucleus will absorb a photon.

8. X-ray imaging sometimes requires the use of contrast dyes. In MRI, dyes are:

A ) less toxic.

B ) not needed.

The dyes used in X-ray imaging must consist of elements from the lower part of the periodic table to take advantage of attenuations being roughly proportional to the atomic numbers of elements contained in the tissue. In contrast, the passage states that adjacent locations with different chemical compositions, when imaged by MRI, already have contrasts of several hundred percent. This implies that a dye is not necessary. Thus, B is the best answer.

C ) needed but not always used.

D ) always used.

9. Human proteins are composed mostly of the elements C, H, O, N, and S. Without dyes, X-ray images of tissue containing

different proteins do NOT have high contrasts, most likely because:

A dyes do not bind well to proteins.

B protein bonds are broken by the radiation, resulting in the formation of free radicals.

Page 3: AAMC MCAT Test 8 A

C the differences in the atomic numbers of the elements are not large enough.

Because attenuation is roughly proportional to the atomic numbers of elements contained in the tissue, to see significant differences in the X-ray image one needs a sizable difference in atomic number. Noting that bones, but not much else, easily appear in an X ray of undyed tissue, one can infer that Ca has an atomic number high enough to significantly attenuate X rays. Of the elements C, H, O, N, and S, only sulfur has an atomic number approaching (but still less than) that of Ca. Thus, C is the best answer.

D proteins are opaque to X rays.

10. For a given magnitude of B1, the nucleus with the nonzero precession frequency will be which of the following?

A 42

He

B 16 8

O

C 19 9

F

Because protons and neutrons have spin just as electrons do, to guarantee a nonzero net spin, an odd number of nucleons is needed. Of the foils, the only element that has an odd number of nucleons is. Thus, C is the best answer.

D 208 82

Pb

11. To adjust ωd of H nuclei, a diagnostician is most likely to vary which of the following?

A ) B1

Based on the passage, the angular frequency ωd is

One can see that B1 is the only externally controllable variable. Thus, A is the best answer.

B ) B3

C ) h

D ) µ

12. According to the passage, at resonance, B2 rotates an H nucleus through an angle of:

A ) 45°.

B ) 90°.

C ) 180°.

The passage states that to precess µ at a frequency of ωd, µ must be parallel to B1. If the precesssing H atom becomes antiparallel, this means µ has turned by 180°. Thus, C is the best answer.

D ) 270°.

13. The effect of temperature on the rate of the reaction can best be determined by comparing Tube 5 with which of the

following tubes?

A ) Tube 2

B ) Tube 3

C ) Tube 4

To determine the effect of temperature on the rate of the reaction, Tube 5 must be compared with another tube in which all conditions were the same, except the temperature. Tube 5 contained 20 mL each of solutions A and B and no water at 32ºC. Tube 4 contained 20 mL each of solutions A and B and no water at 12ºC. Thus, C is the best answer.

D ) Tube 6

Page 4: AAMC MCAT Test 8 A

14. The results in Table 1 would most likely NOT be affected if the students had added excess:

A ) KI(aq) to Solution A.

B ) Na2S2O3(aq) to Solution A.

C ) (NH4)2S2O8(aq) to Solution B.

D ) starch to Solution B.

Starch is not a participant in any of the reactions, acting only as an indicator of excess I2. Thus, D is the best answer. 15. In Tube 6, what is the most likely function of CuSO4(aq)?

A ) Reactant

B ) Indicator

C ) Inhibitor

D ) Catalyst

A comparison of the results for Tube 1 and Tube 6, which are identical except for the addition of CuSO4 to Tube 6, shows that it takes longer for the combined solution in Tube 1 to turn dark blue than it takes for the combined solution in Tube 6. Thus, D is the best answer.

16. The solution in Tube 1 turned dark blue more rapidly than did the solution in Tube 4, because the:

A ) rate of Reaction 2 was slower in Tube 1 than in Tube 4.

B ) average kinetic energies of I(aq) and S2O8

(aq) were greater in Tube 1 than in Tube 4. The reaction conditions for tubes 1 and 4 are identical except Tube 1 was run at 22ºC and Tube 4 was run at 12ºC. The higher the temperature, the greater the average kinetic energy of the components of the reaction. Thus, B is the best answer.

C ) concentrations of I(aq) and S2O8

(aq) were greater in Tube 1 than in Tube 4.

D ) concentration of starch was greater in Tube 1 than in Tube 4.

17. Which of the following graphs best shows the number of moles of S4O6

2(aq) in Tube 6 as time passes?

A )

B )

The data in Table 1 show that the reaction is complete in 19 sec. The amount of S4O62

would increase for 19 sec and then level off. Thus, B is the best answer.

C )

Page 5: AAMC MCAT Test 8 A

D )

18. According to the results of Galileos experiments described in the passage, the ratio of d to t2 attained its maximum value

when:

A ) the inclined plane was horizontal.

B ) the inclined plane was vertical.

Because the acceleration of gravity is in the vertical direction, the greatest acceleration of a sphere will occur when the plane is also vertical. Thus, B is the best answer.

C ) natural motion began.

D ) violent motion ceased.

19. As described in the passage, Aristotles theories of motion do NOT deal with which of the following observations?

A ) An object in a vacuum can move with constant velocity.

Of the foils, the only situation that does not occur naturally on Earths surface is that of an object in a vacuum. Thus, Ais the best answer.

B ) An object in free fall can reach a constant velocity.

C ) A mass on a spring undergoes violent motion.

D ) A heavy body falls with a greater velocity than a light one.

20. When designing his experiment, Galileo could have allowed spheres to drop from a height of 10 m rather than using the 10-

m inclined plane described in the passage. The main advantage to using the inclined plane is that on the inclined plane the:

A ) final velocity of a sphere is smaller.

B ) final velocity of a sphere is larger.

C ) spheres take longer to reach the bottom.

Ignoring air resistance, rotation, and friction, the final velocity of a sphere that drops is the same as the final velocity of a sphere that moves down the inclined plane. Meanwhile, the path is longer on the inclined plane than in free fall, leading to a longer time for a sphere to reach the bottom. Thus, C is the best answer.

D ) spheres take less time to reach the bottom.

21. Spheres A, B, C, and D have identical radii and masses of 10 g, 40 g, 20 g, and 30 g, respectively. According to Aristotles

theory, if the spheres are dropped to the ground from the same height, which object will have the highest terminal velocity?

A ) Object A

B ) Object B

Aristotles theory states that an objects time of fall t is inversely proportional to its weight W. Symbolically, this is t ∝ 1/W. The question asks one to determine the highest terminal velocity (when striking the ground). The highest terminal velocity should then be inversely proportional to t because spheres AD all fall from the same height. Because the largest mass in this group is Sphere B, its weight will also be the largest, so Aristotles theory predicts the shortest t for it. Thus, B is the best answer.

C ) Object C

D ) Object D

22. Object A has 2 times the density and ½ the volume of Object B. According to Aristotles theory, if A and B are dropped to

Earth from the same height, which object will reach Earth first? (Note: Ignore the effects of friction.)

A ) Object A, because it is denser than object B

Page 6: AAMC MCAT Test 8 A

B ) Object A, because it is heavier than object B

C ) Object B, because it is heavier than object A

D ) The objects will reach Earth at the same time.

Object A is given as having twice the density of Object B (ρA = 2ρB) and having half of Object Bs volume (VA = VB/2). To find which object has the shorter time of fall under Aristotles theory (t ∝ 1/W), one must determine the weight of each object from its mass. Because W = mg and m = ρV,

WB = mBg = ρBVBg, and

WA = mAg =ρAVAg = (2ρB)(VB/2)g = ρBVBg

This means that WA = WB, and the time of fall for each object will be the same. Thus, D is the best answer. 23. A sparingly soluble metal hydroxide, M(OH)2 has a molar solubility of S mol/L at 25°C. Its Ksp value is:

A ) S2.

B ) 2S2.

C ) 2S3.

D ) 4S3.

The Ksp for a substance, AaBb, equals [A]a[B]b. The Ksp for M(OH)2 = [M][OH]2. If the solubility of M(OH)2 is S mol/L, then [M] = S mol/L and [OH] = 2S mol/L. The Ksp = S(2S)2 = S(4S2) = 4S3. Thus, D is the best answer.

24. If the quantity of gas remains unchanged while its temperature increases, the volume of the gas will:

A ) always increase, because volume is directly proportional to temperature.

B ) always decrease, because volume is inversely proportional to temperature.

C ) increase if pressure remains constant.

The ideal gas law shows that PV = nRT. If n (the quantity of gas) is constant and T (temperature) increases, then PV (pressure × volume) will increase. If the pressure remains constant, then the volume must increase. Thus, C is the best answer.

D ) decrease if pressure remains constant.

25. An astronaut on Earth notes that in her soft drink an ice cube floats with 9/10 of its volume submerged. If she were instead

in a lunar module parked on the Moon where the gravitation force is 1/6 that of Earth, the ice in the same soft drink would float:

A ) with more than 9/10 submerged.

B ) with 9/10 submerged.

The floating ice cube implies that its weight is balanced by the buoyant force on it

Wice = mg = ρfluidVsubmergedg

Note that both the weight and the buoyant force are proportional to g, making the numerical value of g irrelevant to the volume of the ice cube that is submerged. Thus, B is the best answer.

C ) with 6/10 submerged.

D ) totally submerged.

26. A glass fiber carries a light digital signal long distances with a minimum loss of amplitude. What optical property of glass

allows this phenomenon?

Page 7: AAMC MCAT Test 8 A

A ) Dispersion

B ) Refraction

C ) Reflection Light can be carried along a distance within a transparent material by means of total internal reflection.

Thus, C is the best answer.

D ) Diffraction

27. Reaction 1 is an effective means of decreasing the hardness of an acidic water sample because it:

A ) increases the pH and decreases the solubility of CaCO3.

According to the passage, calcium ions in the water contribute to the hardness of the water and CaCO3 is soluble in acidic solutions. Boiling the water causes Reaction 1 to take place; the solubility of CO2 decreases as the temperature increases. Reaction 1 decreases the hardness of an acidic water sample by removing CaCO3 from the sample and increasing the pH by reducing the amount of CO2 in the sample. Thus, A is the best answer.

B ) decreases the pH and decreases the solubility of CaCO3.

C ) increases the pH and converts HCO3

to water.

D ) decreases the pH and removes Ca2+ from solution.

28. What is the effect of a high level of atmospheric CO2 on the hardness of water in limestone regions?

A ) It would lower the pH and increase the solubility of CaCO3.

High levels of CO2 in the atmosphere would result in CO2 dissolved in the water. When CO2 dissolves in water it becomes carbonic acid, causing a decrease in the pH. The passage states that CaCO3 is very soluble in acidic solutions. The pH would be lowered and the solubility of CaCO3 would increase. Thus, A is the best answer.

B ) It would raise the pH and increase the solubility of CaCO3.

C ) It would lower the pH and decrease the solubility of CaCO3.

D ) It would raise the pH and decrease the solubility of CaCO3.

Solution

Guess

29. How many grams of CaCO3 would be formed in Reaction 1 if the CO2 evolved occupies 11.2 L at STP?

A ) 0.5 g

B ) 11.2 g

C ) 50 g

At STP, 1 mole of gas occupies 22.4 L; therefore, 0.5 mol CO2 was generated. According to Reaction 1, 1 mol CaCO3 forms for every mole of CO2 that forms. There is 0.5 mol CaCO3, which has a formula weight of 100 g/mol. The amount of CaCO3 formed is 50 g. Thus, C is the best answer.

D ) 100 g

Solution

Guess

30. If the pH of the water sample were high such that all the carbonate is present as CO32, what would be the

concentration of Ca2+? (The Ksp of CaCO3 is 4.8 × 109.)

A ) (4.8 × 109)2 M

B ) (4.8 × 109)1/2 M

The Ksp of CaCO3 = [Ca2+][CO32] = 4.8 × 109. If all of the carbonate is in the form of CO3

2, then [Ca2+] = [CO3

2] = (4.8 × 109)1/2 M. Thus, B is the best answer.

C ) 4.8 × 109 M

D ) (4.8 × 109)1/3 M

31. The addition of excess Ca2+ to a solution containing Ca2+ and CO3

2 ions causes CaCO3 to precipitate because:

A ) Ksp for CaCO3 would increase due to the increase in [Ca2+].

Page 8: AAMC MCAT Test 8 A

Solution

Guess

B ) Ksp for CaCO3 would decrease due to the increase in [Ca2+].

C ) [CO3

2] would increase to maintain Ksp.

D ) [CO3

2] would decrease to maintain Ksp.

When CaCO3 is in solution, the following takes place: CaCO3(s) ↔ Ca2+(aq) + CO32(aq) and Ksp =

[Ca2+][CO32]. When excess Ca2+ is added, some CaCO3 precipitates in accordance with Le Châteliers

principle. [CO32] therefore decreases and the Ksp is maintained. Thus, D is the best answer.

32. What is the expression for the angular frequency ω of a pendulum?

A ) 2πmg/L

B ) (L/g)1/2

C ) 2π(g/L)1/2

D ) (g/L)1/2

Because the frequency f of a pendulum in Hz is

the angular frequency ω (in rad/s) is

Thus, D is the best answer.

Solution

Guess

33. The equation for tension used in the passage requires θo to be units of radians. How is an angle of 30° converted to radians?

A ) θo = 30° × π

B ) θo = 30° × 2π

C ) θo = 30° × π/180°

Because 2π rad = 360°, the appropriate conversion factor for degrees-to-radians is π rad/180°. Thus, C is the best answer.

D ) θo = 30° × π/360°

Page 9: AAMC MCAT Test 8 A

Solution

Guess

35. What is the magnitude of the restoring force on the pendulum bob at angle θ?

A ) mg

B ) mgsinθ

The restoring force on a pendulum points along the tangent to the circular arc of its motion. Figure 1 shows the free-body diagram of the pendulum bob, and the only force acting along the direction of motion is mgsin θ. Thus, B is the best answer

C ) mgcosθ

D ) mgtanθ

Solution

Guess

36. What mechanism supplies the tension in the string at the molecular level?

A ) Magnetic forces

B ) Electron transfer

C ) Gravitational forces

D ) Stretching bond lengths

Intermolecular forces and the physical structure of the molecules determine the structure of the string at the molecular level. Physically stretching the string affects the intermolecular forces (which are primarily electrostatic) and the bond lengths within the molecules. Thus, D is the best answer.

37. Each lead weight has a volume of 4 × 106 m3. What buoyancy force does the water exert on a lead weight?

A ) 4.0 × 102 N The buoyancy force FB is

Thus, A is the best answer.

B ) 4.5 × 102 N

C ) 4.1 × 101 N

D ) 4.5 × 101 N

Solution

Guess

34. If all the resistors in Figure 3 are 200 Ω, what is the current from the battery when V0 = 12 V?

A ) 30 mA

B ) 60 mA

There are two branches to this parallel circuit: one passes through R1 and R2 in series, and the other passes through R3 and RSG in series. The total resistance of the parallel circuit is then

Given that R1, R2, R3, and RSG are each 200 Ω, Rtotal is also 200 Ω, and the current Itotal = V/Rtotal is then 0.060 A. Thus, B is the best answer.

C ) 120 mA

D ) 240 mA

Page 10: AAMC MCAT Test 8 A

38. Which of the following items of information would NOT help in predicting the results shown in Figure 1?

A ) The number of air molecules inside the balloon

B ) The thermal conductivity of the rubber

C ) The variation with depth in the speed of the balloon

D ) The total mass of the water in the tank

The mass of the water in the tank will not have an effect on the temperature of the air or the depth of the balloon. Thus, D is the best answer.

Solution

Guess

39. Which of the following statements best explains the temperature change shown in Figure 1?

A ) The work done on the gas by the water pressure decreases its temperature.

B ) The work done by the gas in expanding decreases its internal energy.

The key to understanding the correct statement is to realize that in Figure 1, depth increases to the right. This means that air temperature in a rising balloon will follow the plotted curve from right to left, and temperature falls as the balloon rises. The true statement must reflect the decreasing temperature and pressure in the balloon as it rises. Thus, B is the best answer.

C ) The balloon and water exchange heat, increasing the temperature of the gas.

D ) The compression of the gas decreases its temperature.

Solution

Guess

40. Air-filled, thin-walled steel spheres were suggested for experiments in the tank instead of rubber balloons. The most likely reason that this idea was rejected is that steel spheres would:

A ) not be sufficiently flexible.

According to the passage, the tank is used to study the effects of pressure on gases. If air-filled, thin-walled steel spheres were used, it would be difficult to vary the pressure on the gases. Thus, A is the best answer.

B ) be too heavy.

C ) not be airtight.

D ) rust too quickly.

Solution

Guess

41. What vertical forces are acting on a balloon as it ascends?

A ) Weight only

B ) Buoyancy force only

C ) Weight and buoyancy force only

The foils list only water pressure, buoyancy, and gravity as acting on a balloon. Of these, gravity and buoyancy act in the vertical direction, while water pressure acts perpendicularly to all surfaces of the balloon. Thus, C is the best answer.

D ) Water pressure and buoyancy force only

Solution

Guess

42. If the valve is opened to drain the tank, where is the speed of the flowing water the greatest?

A ) At the top of the tank

B ) At the bottom of the tank

C ) At the wide end of the pipe

D ) At the narrow end of the pipe

For a given volume flow rate, the speed of fluid flow is inversely proportional to the cross-sectional area through which the fluid flows. Thus, D is the best answer.

Page 11: AAMC MCAT Test 8 A

Solution

Guess

43. A balloon is attached to a weight that keeps it from ascending quickly enough for the air in the balloon to change temperature. When the volume of the balloon has doubled, how has the pressure of the air inside changed?

A ) It has been reduced to one-quarter the original pressure.

B ) It has been reduced to one-half the original pressure.

The amount of air in the balloon and the temperature of the air will remain constant. Using the ideal gas law, nRT = P1V1 = P2V2. If the volume of the balloon doubles, then V2 = 2V1, P1V1 = P2 × 2V1, and P2 = P1/2. The pressure therefore reduces by 1/2. Thus, B is the best answer.

C ) It has remained at the original pressure.

D ) It has been increased to twice the original pressure.

44. The process taking place at the cathode was:

A ) oxidation by a loss of electrons.

B ) oxidation by a gain of electrons.

C ) reduction by a loss of electrons.

D ) reduction by a gain of electrons.

According to the passage, H2(g) was produced at the cathode. The reaction taking place at the cathode was 2H+(aq) + 2e → H2(g), which means the H+ was gaining electrons and undergoing reduction. Thus, D is the best answer.

Solution

Guess

45. What is the electron configuration of the metal ion produced when Ca reacted with water in Experiment 1?

A ) 1s22s22p63s23p64s2

B ) 1s22s22p63s23p64s1

C ) 1s22s22p63s23p6

When Ca is added to water, the following reaction takes place: Ca(s) + 2H2O(l) → Ca2+(aq) + 2OH(aq) + H2(g), producing a basic solution. The metal ion produced is Ca2+. Ca has the atomic number 20 and therefore has 20 electrons. Two electrons are removed to form Ca2+, so Ca2+ has 18 electrons. The electron configuration for Ca2+ is 1s22s22p63s23p6. Thus, C is the best answer.

D ) 1s22s22p63s23p64s23d2

Solution

Guess

46. Which of the following properties is most useful in explaining the trend in the reactivities in Experiment 1?

A ) Electronegativity

B ) Ionization potential

The metals that reacted in Experiment 1 underwent oxidation. The energy required to remove an electron from an atom is the ionization potential. The reactivity of a metal depends on its ionization potential. Among the metals listed in Table 1, potassium has the smallest ionization potential and magnesium has the largest ionization potential. Therefore, potassium reacted the most vigorously and there was no obvious reaction with magnesium. Thus, B is the best answer.

C ) Electron affinity

D ) Polarizability

Solution

Guess

47. Experiment 1 was repeated with 0.40 g of calcium, and the gas that evolved was collected. The identity of the gas, and its approximate volume at 1.0 atm and 27°C were: (Note: R = 0.0821 Latm/molK)

A ) H2, 250 mL.

Calcium undergoes the following reaction with water: Ca(s) + 2H2O(l) → Ca2+(aq) + 2OH(aq) + H2(g). The gas produced was H2. If 0.40 g calcium reacted, then the number of moles of calcium reacted was equal to (0.40 g)/(40.1 g/mol) = 0.01 mol. The amount of H2 formed was also 0.01 mol. At 1.0 atm and 27°C, the volume of 0.01 mol H2 = [(0.01 mol)(0.0821 L⋅atm/mol⋅K)(300 K)]/1.0 atm = 0.246 L = 246 mL. Thus, A is

Page 12: AAMC MCAT Test 8 A

the best answer.

B ) H2, 500 mL.

C ) O2, 250 mL.

D ) O2, 500 mL.

Solution

Guess

48. When alkali metals are held in a colorless flame, the metals vaporize and impart a brilliant color to the flame. Lithium imparts a bright red color, sodium a yellow color, and potassium a violet color. What is the origin of these colors?

A ) Excited electrons in the metals drop to lower energy levels and emit specific wavelengths of light.

When alkali metals are held in a flame, the electrons in the atoms are excited to higher energy levels. As the excited electrons drop to lower energy levels, they emit specific wavelengths of light. Thus, A is the best answer.

B ) Electrons in the metals are raised to higher energy levels by absorbing specific wavelengths of light.

C ) Electrons in the metals are converted into gases in the flame that emit specific wavelengths of light.

D ) Specific wavelengths of light are absorbed when the metal is converted from the solid phase to the gas phase

in the flame. 49. Consider the phase diagram of water below.

The arrow proceeding from Point A to Point B represents:

A ) melting.

B ) sublimation.

C ) condensation.

D ) vaporization.

In the phase diagram, Point A is in a region where water exists as a liquid and Point B is in a region where water exists as a vapor. The arrow proceeding from Point A to Point B represents the transition from liquid to vapor, or vaporization. Thus, D is the best answer.

50. Which of the following statements best characterizes a material that is a good insulator but a poor conductor?

A ) The material contains no electrons.

B ) The magnitude of the electric field inside the material is always equal to zero.

C ) The atoms in the material can easily move from one lattice site to another.

D ) Electrons in the material cannot easily move from one atom to another.

In insulators, the valence electrons are tightly bound to their atoms, and it takes a great amount of energy to free them from their atomic energy levels. Thus, D is the best answer.

Page 13: AAMC MCAT Test 8 A

Solution

Guess

51. An object with a mass of 0.1 kg absorbs 250 J of heat while changing temperature from 20°C to 25°C. What is the specific heat of the object?

A ) 125 J/(kg°C)

B ) 250 J/(kg°C)

C ) 375 J/(kg°C)

D ) 500 J/(kg°C)

Heating that produces only a temperature change obeys the relationship

Given 250 J of heat absorbed by the objects mass of 0.1 kg, the temperature change of 5°C implies a specific heat of about 500 J/(kg•°C). Thus, D is the best answer.

Solution

Guess

52. At a given temperature, the resistance of a wire to direct current depends only on the:

A ) voltage applied across the wire.

B ) resistivity, length, and cross-sectional area.

The resistivity relation at a fixed temperature is

Thus, B is the best answer.

C ) inductance, length, and cross-sectional area.

D ) resistivity, length, and capacitance.

Solution

Guess

53. Which of the following electron configurations of nitrogen is the most stable?

A ) 1s22s12p4

B ) 1s22s22p3

The atomic number of nitrogen is 7. The electrons will begin to fill the lower energy levels first; consequently, the 1s level will fill first with two electrons, followed by 2s with two electrons, and then the 2plevel with the last three electrons. The resulting electron configuration is 1s22s22p3. Thus, B is the best answer.

C ) 1s12s22p4

D ) 1s22s22p23s

54. How many sodium ions are in the initial 50.00-mL solution of Na2CO3?

A ) 1.50 × 1022

B ) 3.00 × 1022

The solution contains 7.15 g Na2CO3⋅10H2O. Dividing by the molar mass will give the number of moles: (7.15 g)/(286.14 g/mol) = 0.0250 mol. Because each mole of Na2CO3⋅10H2O contains 2 mol Na+, there is 0.0500 mol Na+. Using Avogadros number, the number of sodium ions is (0.0500 mol)(6.02 × 1023 ions/mol) = 3.01 × 1022 ions. Thus, B is the best answer.

C ) 6.02 × 1023

D ) 12.0 × 1023

Page 14: AAMC MCAT Test 8 A

55. If red litmus paper is dipped into the Na2CO3 solution, it will:

A ) remain red, because carbonate is an acidic salt.

B ) remain red, because sodium carbonate is neutral.

C ) turn blue, because carbonate reacts with water to produce OH.

In water, carbonate will undergo the following reaction: CO32(aq) + H2O(l) → HCO3(aq) + OH(aq). Red litmus

paper will turn blue in a base. Thus, C is the best answer.

D ) turn blue, because sodium ions form sodium hydroxide in water.

Solution

Guess

56. What is the maximum number of moles of nickel carbonate (NiCO3) that can form during the precipitation reaction?

A ) 0.025 mol

There is 0.025 mol Ni from NiSO4⋅6H2O: (6.57 g)/(262.84 g/mol) = 0.025 mol. There is also 0.025 mol CO3

2 from Na2CO3⋅10H2O: (7.15 g)/(286.14 g/mol) = 0.0250 mol. The maximum number of moles of NiCO3 that can form is 0.025 mol. Thus, A is the best answer.

B ) 0.25 mol

C ) 1.00 mol

D ) 2.25 mol

Solution

Guess

57. According to the information in the passage, the gas that evolves is:

A ) sulfur dioxide.

B ) sulfur trioxide.

C ) carbon dioxide.

According to the passage, the gas is given off when HCl is added to one portion of the slurry. HCl reacts with CO3

2 to form CO2(g). Thus, C is the best answer.

D ) carbon monoxide.

Solution

Guess

58. The ion responsible for the color of the solutions is:

A ) sulfate, because sulfur has s and p orbitals.

B ) nickel(II), because it has a charge of +2.

C ) nickel(II), because it has unfilled d orbitals.

The solution containing nickel(II) ions is green-colored. The color arises because nickel(II) ion has partially filled d orbitals and the electrons in the lower energy d orbitals absorb visible light to move to the higher energy d orbitals. Thus, C is the best answer.

D ) sulfate, because it is a resonance-stabilized anion.

59. Suppose a scientist tests Metals K, L, and M for use in Plate A of the photoelectric experiment. The work functions of K, L,

and M are 5.0 eV, 8.0 eV, and 6.0 eV, respectively. If each metal is struck by a 7.0 eV photon, what will be the maximum kinetic energies (in eV) of the electrons ejected, if any, from K, L, and M, respectively?

A ) 2.0, 0.0, 1.0

The maximum kinetic energy of photoelectrons is always E − W. Note that because Metal L has WL > E, the maximum K = 0 (there are no photoelectrons). Thus, A is the best answer.

Page 15: AAMC MCAT Test 8 A

B ) 2.0, 1.0, 1.0

C ) 2.0, 15.0, 0.0

D ) 12.0, 15.0, 13.0

Solution

Guess

60. In the photoelectric experiment, which of the following changes will produce the largest increase in the value for I?

A ) Plate A is replaced with one having a higher work function.

B ) The electrical potential of Plate B is made more positive relative to that of Plate A.

The current I will be directly proportional to the number of photoelectrons leaving the metal at Plate A. A photoelectron that leaves Plate A will only be counted among the current I if its value of K is greater than the stopping potential. Foil B describes a change that reduces the stopping potential, allowing more photoelectrons to reach Plate B. Thus, B is the best answer.

C ) The flux of photons having energies less than W is increased.

D ) The average frequency of the photons is decreased.

Solution

Guess

61. In Youngs experiment, if wavelength is increased, the bright fringes will become:

A ) brighter.

B ) bluer.

C ) closer together.

D ) farther apart.

In Youngs double-slit experiment, bright fringes appear on the screen as the column of dark spots in the following figure.

For a bright fringe to appear, the difference in distances is equal to an integral number of wavelengths, and the distance from one fringe to the next on the screen reflects a change in the value of

that is directly proportional to one wavelength. Thus, D is the best answer.

Solution

Guess

62. If light is wavelike and if diffractive effects are ignored, the fringes on T3 in Figure 1 should disappear when:

A ) λ is increased slightly.

B ) light intensity is increased.

C ) S1 is covered.

D ) T3 is moved farther from T2.

Page 16: AAMC MCAT Test 8 A

D ) T3 is moved farther from T2.

Solution

Guess

63. In Youngs experiment, S produces spherical wave fronts. The light entering S1 and S2 will most likely consist of plane waves if the distance between:

A ) T1 and T2 is large.

Because wavefronts from a spherical source become flatter and flatter as they move away from their source, increasing the distance between T1 and T2 will cause waves from S to become more planar. Thus, A is the best answer.

B ) T1 and T2 is small.

C ) S1 and S2 is large.

D ) T2 and T3 is small.

Solution

Guess

64. When Waves A, B, and C meet at the same point on the target screen, Wave A is 180° out of phase with Wave B and 0° out of phase with Wave C. If each of the 3 waves has an amplitude E when it reaches this point, the resulting wave amplitude at the point will be:

A ) zero.

B ) E.

The principle of superposition allows one to add incoming waves amplitudes together to determine the total amplitude at a point. Based on the passage, if Wave A has amplitude +E, the amplitudes of waves B and C will be −E and +E, respectively, and the sum will be +E. Thus, B is the best answer.

C ) 2E.

D ) 3E.

Solution

Guess

65. The wave theory states that for an electron to be ejected from a metal, light of sufficient intensity must be used. Based on the passage, does the particle theory agree with this aspect of the wave theory?

A ) Yes, because light intensity determines the work function of a metal

B ) Yes, because light intensity depends only on frequency

C ) No, because light waves cannot carry sufficient energy to eject an electron

D ) No, because an electron can be ejected by a single high-energy photon

At the core of the photoelectric effect is the concept that even a single photon can release a photoelectron, but only if that photon carries enough energy. Intensity in the photon picture is a measure of how many photons are arriving each second, not how much energy each photon has. This means that photoelectrons can be released even when only a few high-energy photons are present. If, however, the photon energy is too low, even a flood of photons will not release any photoelectrons. Thus, D is the best answer.

66. If Solution A contains Ag+, the anion component must be:

A ) CrO4

2.

B ) Cl.

C ) F.

According to the passage, the ionic compound in Solution A was completely soluble in water. The information in Table 1 shows that the only Ag+ salt studied that is soluble in water is AgF. Thus, C is the best answer.

D ) S2.

67. When aqueous solutions of the various anions and cations were mixed, precipitates formed because:

A ) few aqueous solutions can contain more than one cation or anion.

B ) the anions precipitated as solid metals.

C ) the solubilities of cations were decreased by the other cations.

Page 17: AAMC MCAT Test 8 A

C ) the solubilities of cations were decreased by the other cations.

D ) the solubility product of a compound was exceeded.

The amount of a substance that will dissolve in water is described by the Ksp. The Ksp for a substance, AaBb, equals [A]a[B]b. If the amount of the compound present is in excess of the Ksp, then a precipitate would form to maintain the Ksp. Thus, D is the best answer.

68. All of the precipitation reactions in Table 1 can be classified as:

A ) metathesis reactions.

The precipitation reactions in Table 1 are the result of mixing aqueous solutions of cations and anions. The ionic compounds switch the ions with which they are paired. These are reactions of the type AB + CD → AD + CB, where one of the products is a precipitate. This type of reaction is a metathesis reaction. Thus, A is the best answer.

B ) decomposition reactions.

C ) hydrolysis reactions.

D ) oxidation reactions.

Solution

Guess

69. In Table 1, which cation allowed for the greatest number of soluble compounds?

A ) Ag+

B ) Ca2+

According to Table 1, Ag+ had one soluble compound, Ca2+ had three soluble compounds, and Cu2+ and Fe3+ each had two soluble compounds. Ca2+ had the greatest number of soluble compounds. Thus, B is the best answer.

C ) Cu2+

D ) Fe3+

Solution

Guess

70. Ba2+(aq) is an ion that is very toxic to mammals when taken internally. Which of the following compounds, mixed in water, would be the safest if accidentally swallowed?

A ) BaSO4, Ksp = 1.1 × 1010

The lower the value of Ksp is, the lower the concentrations of the cation and anion in an aqueous solution and the lower the solubility of the compound in water. If mixed with water and accidentally swallowed, the Ba salt with the lowest value of Ksp would be the safest. Thus, A is the best answer.

B ) BaCO3, Ksp = 8.1 × 109

C ) BaSO3, Ksp = 8.0 × 107

D ) BaF2, Ksp = 1.7 × 106

Solution

Guess

71. Which one of the following cations or anions would be useful in differentiating between a solution containing FeCl3 and one containing FeF3?

A ) CrO4

2

B ) Ca2+

According to the data in Table 1, CaF2 will form a white precipitate, but CaCl2 is water-soluble. Using Ca2+, one could differentiate between a solution of FeCl3 and FeF3. Thus, B is the best answer.

C ) S2

D ) Cu2+

72. A battery in a circuit has an electromotive force given by E and an internal resistance of r. The battery provides a current i

to the circuit What is the terminal voltage of the battery?

A ) E

Page 18: AAMC MCAT Test 8 A

A ) E

B ) E ir

The terminal voltage is the voltage provided to the external components of the circuit. The battery voltage ¼ will be reduced by the voltage required to overcome the internal resistance, so V = ¼ − ir. Thus, B is the best answer.

C ) E + ir

D ) E + i2r

Solution

Guess

73.

A beam of light shines into a transparent medium with parallel surfaces. Part of the beam is reflected back into the air as diagrammed above. (The figure is NOT to scale.) The index of refraction of the medium is 1.5. Which of the following is true?

A ) θ < θ ′ and θ < α

B ) θ = θ ′ and θ > α

Because the mediums surfaces are parallel, a perpendicular line drawn to the lower surface of the medium will be parallel to both of the perpendiculars shown in the figure. This means that the angle of incidence at the lower surface will also be α, as will the angle of reflection at the lower surface, and the beam reflecting from the lower surface of the medium will then be a mirror image of the incoming beam, so θ′ = θ. Further, because airs index of refraction is about 1.0, Snells law would show that θ > α. Thus, B is the best answer.

C ) θ = θ ′ and θ < α

D ) θ > θ ′ and θ > α

Solution

Guess

74. Although only molecules having much greater than the average kinetic energy can escape from a liquid, the temperature of a liquid in equilibrium with its vapor is found to be the same as that of the vapor. How can this be interpreted?

A ) The excess kinetic energy is transferred to air molecules.

B ) The excess kinetic energy is lost in collisions with the walls of the container.

C ) The higher kinetic energy of the vapor molecules is compensated for by their lower potential energy.

D ) The excess kinetic energy is expended on overcoming attractive forces exerted by the molecules of the

liquid. If the liquid and the vapor have the same temperature, then the average kinetic energy of the liquid and the vapor must be the same. The molecules that escape from the liquid are those with greater than the average kinetic energy. It takes energy for the molecules to escape from the liquid. Thus, D is the best answer.

Solution

75. Sound of a known frequency, wavelength, intensity, and speed travels through air and bounces off an imperfect reflector which is moving toward the source Which of the following properties of the sound remains the same

A ) Speed

Within still air the speed of sound remains constant Thus A is the best answer

B ) Intensity

C ) Frequency

Page 19: AAMC MCAT Test 8 A

Guess

C ) Frequency

D ) Wavelength

Solution

Guess

76. In order to determine the relative speed of approach of a sound source by Doppler measurements, three of the following items of data are necessary. Which one is NOT required?

A ) The speed of sound in the medium

B ) The frequency of the emitted sound

C ) The frequency of the observed sound

D ) The distance between source and observer

Given that v is the speed of sound in the medium, the Doppler equation for a source that is approaching (receding from) an observer can be written as

Thus, D is the best answer.

Solution

Guess

77. Which of the following is equal to a change in momentum of an object?

A ) Force

B ) Acceleration

C ) Velocity

D ) Impulse

A change in momentum can be accomplished by the action of a force F over a time interval ∆t. This product, F∆t, is referred to as an impulse. Thus, D is the best answer.

78. The assertion that the introduction of an alphabet changes cognitive habits is:

A ) true, on the basis of the low literacy rate in the U.S.

The author makes no reference to the literacy rate in the U.S. or whether it is low or high, so no determination on whether this is true can be made here.

B ) supported by objective data in the passage.

This assertion is made in one sentence without elaboration or objective data. The main body of the passage is devoted to the impact of technology, in particular, television, on culture and does not further discuss the impact of introducing the alphabet.

C ) perhaps true but not explicitly supported by passage information.

This assertion is introduced in the context of the larger point being made in the paragraph about televisions revolutionary impact on society, which was as great, perhaps even greater than introduction of the alphabet. The assertion functions to set up a comparison, so the authors assertion must be assumed to be perhaps true in order for the point made about televisions revolutionary impact to be convincing.

D ) contradicted by the assertion that television watching is pervasive in the U.S.

The assertion contrasts with, but is not contradicted by the passage assertion about the pervasiveness and cultural impact of television watching in the U.S. See rationale C.

Solution

79. Which of the following findings would most weaken the authors argument about the extent to which U.S. society has fulfilled the Huxleyan prophecy?

A ) A high percentage of the U.S. adults who watch television regularly have a good understanding of the

politics and validity of the media.

The author makes a large point about how the television revolution occurred without resistance from a l ti th t thi ki l b li i t h l i l i it bl Th th d thi

Page 20: AAMC MCAT Test 8 A

Guess

point about the unthinking acceptance of television by the people in the final paragraph: Huxley believed that we are in a race between education and disaster, and he wrote continuously about the necessity of our understanding the politics and epistemology of media. . . . he was trying to tell us that what afflicted people in Brave New World was not that they were laughing instead of thinking but that they did not know what they were laughing about and why they had stopped thinking. It stands to reason, then, that the existence of a U.S. television audience that was sophisticated and understood the politics and validity of the media would most challenge and weaken the passage argument.

B ) A high percentage of the U.S. adults who watch television regularly failed to vote in the last presidential

election.

This would not necessarily weaken the argument since this could arguably underscore the authors point about how television has enforced compliance from the people without discussion, opposition, or a vote. It would only follow that, if it were true that television-watching adults were be less inclined to vote, then this would be another instance in which television removed the critical discernment and motivation to engage actively in politics, or even create opposition.

C ) More U.S. adults are able to name the judge on the television show The Peoples Court than are able to

name the U.S. chief justice.

This would prove, not weaken, the authors point about how pervasively television is able to mediate the reality of television-watching adults and remove them from participation in public life. See rationale A.

D ) More U.S. adults have read 1984 than have read Brave New World.

Even if this were true, this would have little effect on the authors conclusions, especially since, unlike the author, U.S. readers would likely not see television culture in light of 1984 or Brave New World. In addition, nothing is said about how many people have read 1984 or Brave New World relative to how many watch television. If the number of readers of these books was much smaller than the number of television watchers, then, even if some of these readers reached the same conclusions as the author, this would have little influence on the culture as a whole.

Solution

Guess

80. The passage suggests that if a news commentator presented an editorial agreeing with the Huxleyan warning, many viewers would:

A ) take whatever action was necessary to combat the danger.

This response would be more likely in the case of an Orwellian culture, which the author states is like a prison and much easier to recognize, and oppose than a Huxleyan [world].

B ) listen carefully to the commentator and then explain the ideas to others.

The passage suggests the opposite response: Huxley believed that we are in a race between education and disaster, and he wrote continuously about the necessity of our understanding the politics and epistemology of media. . . . he was trying to tell us that what afflicted people in Brave New World was not that they were laughing instead of thinking but that they did not know what they were laughing about and why they had stopped thinking. See rationale C.

C ) charge that the commentator was irrational or needlessly alarming viewers.

The passage suggests that the commentator would invite this charge: Those who speak about this matter must often raise their voices to a near hysterical pitch, inviting the charge that they are everything from wimps to public nuisances to Jeremiahs. The author goes on to explain that the reason television critics are compelled to go to such extremes is to call attention to how the Huxleyan world of television culture appears benignthis commentator might be the antidote to a cultures being drained by laughter.

D ) be receptive to learning more about the danger.

The viewers would be unreceptive to learning about the danger, because, according to the author, this world would appear benign. See rationale C.

Solution

Guess

81. One can justifiably infer from the authors argument that if a presidential election campaign in the U.S. involved trivial candidates and discussion, the public would:

A ) vote for the candidates they found to be most trivial.

The public would not be drawn to the most trivial candidate because they would not even notice which one

B ) vote for the candidates they found to be least trivial.

Page 21: AAMC MCAT Test 8 A

According to the author, the public would not be able to discern which candidate was the least trivial. See rationale D.

C ) denounce the entire campaign.

In a Huxleyan world, the public would not even be discerning enough to denounce the campaign, which would be a greater likelihood in an Orwellian world.

D ) not even notice the triviality.

The most justifiable inference is that the public would not even notice based on the authors discussion of how television culture took over without even a pretense of a debate. This also can be inferred from the authors discussion of Brave New World in relation to the Huxleyan world of television culture in the final paragraph: he was trying to tell us that what afflicted people in Brave New World was not that they were laughing instead of thinking but that they did not know what they were laughing about and why they had stopped thinking.

Solution

Guess

82. The author sees the U.S. consuming love affair with television as relevant to Huxleys warning because:

I. television discusses vital matters. II. television is changing peoples way of thinking.

III. technology can cause negative social changes.

A ) I only

References to public business [becoming] a vaudeville act, serious discourse [dissolving] into giggles, and a cultures being drained by laughter suggest that television either does not discuss vital matters or, at best, trivializes them.

B ) II only

Option III is also correct. See rationale D.

C ) I and II only

Option I is incorrect. See rationale A.

D ) II and III only

The author points out how Huxley warns of the negative social consequences of technology: What Huxley teachesis that in the age of advanced technology, spiritual devastation is more likely to come from an enemy with a smiling face than from one whose countenance exudes suspicion and hate. This is then reinforced by a reference to technological distractions. The author later makes an even stronger statement concerning technology and negative social change: To be unaware that technology comes equipped with a program for social change, to maintain that technology is neutral, to make the assumption that technology is always a friend to culture isstupidity plain and simple. The author also underscores the idea that technology is changing the way people think: for it imposes a way of life, a set of relations among people and ideas, about which there has been no consensus, no discussion, and no opposition, only compliance. Public consciousness has not yet assimilated the point that technology is ideology.

Solution

Guess

83. A study concluding that political experts consider the U.S. presidential election a personality contest rather than a clash of issues would:

A ) support the authors point that public business has become another aspect of entertainment.

The author states the seriousness of reducing public business to entertainment: When . . . people become an audience and their public business a vaudeville act, then a nation finds itself at risk; culture death is a clear

B ) support the authors point that no one is warning the U.S. public of the Huxleyan nightmare.

The author does point out that someone is warning of this, but that they are simply not being heard: Those who speak about this matter must often raise their voices to a near hysterical pitch, inviting the charge that

C ) suggest that Orwell was right in saying that Big Brother is watching people.

The author suggests that, in the Huxleyan world, it is the other way around: Big Brother does not watch us, by his choice. We watch him, by ours.

Page 22: AAMC MCAT Test 8 A

C ) suggest that Orwell was right in saying that Big Brother is watching people.

The author suggests that, in the Huxleyan world, it is the other way around: Big Brother does not watch us, by his choice. We watch him, by ours.

D ) suggest that people believe in the inevitability of progress.

Accepting the inevitability of progress has more to do with the authors point about how acceptance of television technology happens almost unnoticed than with the authors point of how public discourse has become entertainment. See rationale A.

84. What is the main idea of the passage?

A ) Those who create literature understand it more completely than do those who only study it.

The author does not imply this, but, rather, points out that these are two separate activities: One is creative, an art; the other, if not precisely a science, is a species of knowledge or of learning. The author then elaborates on this distinction, perhaps even implying that more complete knowledge may come from the study than from the creation of literature: useful as the experience of literary creation is, the task of the student is completely distinct. The student must translate the experience of literature into intellectual terms, assimilate it to a coherent scheme which must be rational if it is to be knowledge.

B ) The methodologies of science and the study of literature have many features in common.

The author makes this point prior to further developing the main idea. See rationale C.

C ) There are valid methods for studying literature that differ from the methods of science.

The author explains how some of the methodologies of science and literary study overlap, but then goes on to say that this has not fulfilled expectations and that literary study has its own valid methods distinct from the natural sciences: It should be simply recognized that there is a difference between the methods and aims of the natural sciences and the humanities.

D ) The achievements of the humanistic disciplines have been obscured by the achievements of the physical sciences.

This is only a minor point the author makes in acknowledging that this may be so, but the author also proposes that the humanities can be resuscitated to a place of greater prominence.

85. According to the passage, the job of the student of literature is to:

I. discover ways to approach literature intellectually. II. separate the rational from the irrational elements in literary works.

III. integrate the experience of literature as art and the analysis of literature as knowledge.

A ) I only

The author defines the task of the literature student as a purely intellectual exercise: The student must translate the experience of literature into intellectual terms, assimilate it to a coherent scheme which must be rational if it is to be knowledge. The author goes on to describe some of the methods literary study shares with science such as causality, collection of facts, and quantitative methods, and name the methods common to most disciplines such as induction and deduction, analysis, synthesis, and comparison.

B ) II only

The author only acknowledges that literature contains irrational elements, but not that the literature student must separate the rational from the irrational. The author implies that the student of literature only must find rational means of explaining literature, which is no different than the task confronting the musicologist or art historian.

C ) III only

The author argues against the need to integrate what the author sees as two distinct activities, literature and literary study.

D ) I and II only

Only Option I is correct. See rationale A.

86. The author suggests that both art history and musicology should be approached:

Page 23: AAMC MCAT Test 8 A

Solution

Guess

A ) with a strict scientific methodology.

The author does not advocate this for musicology or art history. The author suggests that the transfer of scientific methodology to literary study has not fulfilled the expectations with which it was made originally. By extension, the same would hold true for art history and musicology.

B ) only by those who practice the art form.

The passage opens with the author describing this approach as it applies to literature, then arguing against it. By extension, the author would make the same distinction between music and musicology and art and art history.

C ) intellectually, despite their irrational components.

The author discusses the need to approach the study of literature intellectually, despite irrational elements, which is no different than the task facing the art historian or musicologist.

D ) with the understanding that they are essentially inaccessible to rational study.

The author explains that it is possible to approach the study of literature rationally, even if literature contains irrational elements. The author extends this argument to musicology and art history.

Solution

Guess

87. Assume that a scholar is planning an extensive study of the childrens story, Red Riding Hood. The author of the passage probably would say that the most important task to be performed is:

A ) examining the social context in which the story was written.

A discussion of the social context would likely focus on elements surrounding the story without getting inside it. Although useful, this approach would not be what the author advocates because such a study would not translate the experience of the story itself into terms that convey knowledge of the storys literary elements in particular. Arguably, this would be a socio-historical study more than a literary study.

B ) comparing the occurrence of words indicating various concepts (family relationships, food, emotional states,

etc.).

While this approach would illustrate isolated literary elements in relation to isolated concepts, it would be too fragmented and not create the coherent knowledge the author is looking for. See rationale D.

C ) tracing prior literary influences on the structure of the story and its influence on later works.

This would, arguably, be an example of the approach borrowed from evolutionary biology insofar as it would trace the evolution of literature. While the author would not reject that approach, finding it has some validity, the author would not find it entirely satisfactory either. Such a study would mimic science rather than include the valid methods of literary scholarship that are unique to literary study itself. This study would largely focus on matters extraneous to the story itself rather than the experience of the story, which is what the author advocates.

D ) isolating the story elements that explain its enduring popularity.

Isolating the story elements into a coherent argument to the end of explaining the storys popularity would be an example of the kind of approach the author calls for, which is to translate the experience of literature into intellectual terms, assimilate it to a coherent scheme which must be rational if it is to be knowledge.

Solution

Guess

88. The reader can infer from the passage that its author believes that the use of induction and deduction, analysis, A ) appropriate, because such methods are common to all intellectual disciplines.B ) appropriate, because no certainty about a subject is possible unless information about it is obtained C ) inappropriate, because literature, unlike science, includes irrational aspects that cannot be investigated

Page 24: AAMC MCAT Test 8 A

student is able to translate these elements into intellectual terms and into rational, coherent knowledge.

D ) inappropriate, because of the different purposes of the sciences and the humanities.

The author does acknowledge that the methods of literary scholarship are not always those of the natural sciences. However, the author also states that the sciences and literary study share many methodologies in common. See rationale A.

Solution

Guess

89. If the author of the passage wants to get the most convincing evidence for passage assertions, the author should:

A ) investigate the process of creating a literary work.

The author makes a clear distinction between the creation of a literary work and the study of literature and would not find this approach useful.

B ) use the scientific method to study irrational elements in a literary work.

Because the scientific method focuses largely on objective, quantitative methods to the end of producing certainty, this method, arguably, may have difficulty in accounting for irrational elements in a way that the methods and intellectual terms unique to literary scholarship may not. Therefore, this would be at best a limited approach and at worst a failed approach, which would not provide the evidence the author needs in order to support the main argument of the passage.

C ) apply a particular scientific technique to both a literary work and a problem in the natural sciences.

This approach would exclude methods that are unique to literary scholarship, which would fail to support the authors main point. See rationale D.

D ) compare a literary analysis and a scientific analysis of a literary work.

This approach would be most effective, because the author maintains that literary scholarship shares methods in common with scientific analysis but also uses methods unique to literary study itself.

Solution

Guess

90. How could the author best clarify the statement that literature is irrational, or at least contains strongly unrational elements?

A ) By providing definitions of both irrational and unrational

Since unrational is not a part of common usage, and not found in standard dictionaries, the author would need to define it, which would be at best a provisional definition and one lacking authority. The distinction would be better served by a specific instance of an unrational element in a specific literary work. See rationale B.

B ) By giving an example of an unrational element in a specific work of literature

The word unrational does not represent common usage and the author is clearly coining a word here meant to be distinguished from irrational, which is familiar to common usage. Therefore, this word would be best served by a specific example.

C ) By discussing the irrationality of the creative process

This would not explain the implied distinction in degree that the author makes between unrational and irrational.

D ) By adding the explanation, Human behavior is irrational; therefore stories depicting the truth of human

behavior are likely to seem unrealistic.

This statement would not be relevant to the implied distinction the author is making, since it says nothing about what is meant by unrational.

Solution

Guess

91. The passage suggests that the author believes the study of literature to be important because it: A ) shows that the scientific method can be applied to a wide variety of disciplines. B ) helps the student to become more creative.

Page 25: AAMC MCAT Test 8 A

distinct (italics added).

C ) teaches that the truth can be discovered by nonscientific means.

This implies that knowledge and truth, which the author does not discuss, are the same thing. Even if they were the same thing, the author is not simply saying that knowledge (or truth) can be discovered by nonscientific means but, more crucially, that literary study, in particular, offers its own unique form of knowledge.

D ) offers a type of knowledge unavailable from other disciplines.

The author makes a point about the unique knowledge produced by literary studies: Literary scholarship has its own valid methods, which are not always those of the natural sciences but are nevertheless intellectual methods. The author enlarges this discussion to include the unique contributions to knowledge produced by all of the humanities, of which literary study is a part.

Solution

Guess

92. Assume that contemporary literary studies involving a systematic analysis of text have yielded evidence of underlying attitudes that traditional literary criticism had not detected. How does this assumption relate to assertions made in the passage?

I. It is consistent with the assertion that the transfer of the methods of the natural sciences to art has not fulfilled expectations.

II. It constitutes evidence for the assertion that there is a field in which the methodologies of science and literary study overlap.

III. It is inconsistent with the assertion that philosophy, history, and theology have worked out valid methods of knowing that can be modified to apply to the humanities.

A ) I only

The opposite would be true, namely, that this would be an instance in which transfer did fulfill expectations.

B ) II only

The author states that analysis is common to all types of systematic knowledge, including literary study. This achievement of contemporary literary studies would provide demonstrable evidence that this was true.

C ) I and III only

This would only show that systematic analysis can create new knowledge about literature. It does not logically follow that this added knowledge and methodology would contradict the idea that methods unique only to literary studies have been worked outthese methods would still be unique, even with the addition of one of the scientific methods to the field of literary study. See rationale A.

D ) II and III only

Only Option II is correct. See rationales B and C.

Solution

Guess

93. Elsewhere, the author says that literature is no substitute for sociology or politics, nor is it substitute philosophy. This statement agrees most closely with the passage assertion that:

A ) literary works are sometimes studied in relation to economic, social, and political conditions.

This does not assert the uniqueness of literary study, but, rather, where it intersects with other disciplines.

B ) literary study has its own unique justifications and aims.

The author states: Literary scholarship has its own valid methods. This states the authors main point that, while literary study shares methodologies with other disciplines, it also produces its own unique knowledge.

C ) unlike sociology, politics, or philosophy, literature is an artistic pursuit.

The author would say, instead, that literary study is an intellectual discipline, not an artistic endeavor.

D ) the methodologies of science and literary study often overlap.

The author takes care to point out that, while there is this overlap, literary study offers methods unique unto itself.

Page 26: AAMC MCAT Test 8 A

94. According to the passage, the application of the concept of natural selection to extraterrestrials could be disquieting in its suggestion that:

A ) the most intelligent beings may also be the most aggressive.

The author cites James Trefils caution that if evolution functions approximately the same way on other worlds that it has functioned hereadvanced extraterrestrials might still be aggressive, territorial, and quick to reach for the sword. The most disquieting aspect of natural selectionis that it channels intellect to predators.

B ) not all selection pressures favor predation.

This is an alternative, less disquieting speculation offered in the passage.

C ) extraterrestrials are intellectually superior to humans.

Although it is implied that humans might have something to learn from intelligent extraterrestrials, the actual intellectual superiority of aliens to humans is a speculation never raised in the passage, much less viewed as a disquieting notion.

D ) extraterrestrials may exhibit many recognizable human traits.

The author suggests that this would be a comforting rather than disquieting notion, for it would imply that human nature was something deeper even than we know. Aliens might exhibit many recognizable traits: curiosity, desire for companionship, love of laughter, pleasure in art and culture, and respect for the sanctity of life.

Solution

Guess

95. According to the passage, speculation about the nature of extraterrestrials would be most crucial to humans in a situation in which:

A ) extraterrestrials landed unannounced on Earth.

The author suggests that humans would not have a choice in this case.

B ) humans had the choice of whether to reply to an extraterrestrial contact.

This point is made explicitly in the passage: if alien contact occurred by radio, humanity would face the momentous choice of whether to reply. The decision could turn on speculation about what the other beings were like. Speculation about the nature of the aliens would be even more crucial prior to contacting them, since the option would still exist to avoid them altogether if humans became uncomfortable in their speculationsthe wrong decision in this case could have potentially horrible consequences.

C ) extraterrestrials were hostile to humans.

If the aliens were openly hostile, then it logically follows that there would be no need to speculate about their nature, since that would be obvious.

D ) humans could acquire nuclear knowledge without risking self-destruction.

This would only be a crucial question only if aliens had actually been contacted and the circumstances merited it, namely, that the aliens had actually acquired nuclear knowledge and found nondestructive uses for it, which the author assumes is likely. However, the most crucial situation would be one in which humans had to choose whether or not to contact them. See rationale B.

Solution

Guess

96. In stating that alien civilizations might avoid self-destruction through abhorrent means, the author apparently intends to counter the views of:

A ) James Trefil.

James Trefil suggests that the same evolutionary process that made humans aggressive might make aliens aggressive.

B ) science-fiction writers.

The passage indicates that science fiction writers tend to depict aliens as warlike, a view that is countered by some researchers.

C ) the researcher quoted in the second paragraph.

The researchers optimistic outlook on aliens as no longer menacing is based on the assumption that they have acquired the wisdom to control war. However, this outlook overlooks the possibility that self-destruction may have been avoided through means abhorrent as the author suggests might be a possibility.

Page 27: AAMC MCAT Test 8 A

D ) military tacticians.

The view of military tacticians in regard to aliens is not represented in the passage. The only reference to military tacticians is an analogy between their view of pacifists and how aliens might view mutually assured destruction.

Solution

Guess

97. According to the author, one comforting aspect of the concept of natural selection in reference to extraterrestrials is the possibility that any extraterrestrials that contacted Earth would:

A ) represent the fittest of their species.

This could be disquieting rather than comforting, especially if, as the author suggests in citing James Trefil, this leads to aliens being selected for their aggressive traits.

B ) exhibit recognizably human traits.

Such a view is represented as comforting in the passage because aliens would also exhibit human traits other than aggression: Aliens might exhibit many recognizable traits: curiosity, desire for companionship, love of laughter, pleasure in art and culture, and respect for the sanctity of life.

C ) counter the stereotypes of extraterrestrials promoted by science fiction.

The passage suggests that the stereotypes of aliens in science fiction reflect aggressive traits, although in comical ways. The author finds the possibility that they may be aggressive disquieting. See rationale A.

D ) explain to us their means of avoiding nuclear destruction.

In this case, the author is also disquieted by the possibility that such aliens might avoid self-destruction by means abhorrent.

Solution

Guess

98. Given the information in the passage, one would expect the nature of an alien civilization that contacted Earth to be:

A ) either warlike or pacifistic.

The passage devotes nearly equal time to both possibilitiesspeculation that aliens may be pacific and speculation that they may be warlike.

B ) warlike by definition.

The author suggests that aliens could be just as easily pacific as warlike. See rationale A.

C ) pacifistic by definition.

The passage argues for the equal possibility that they may be warlike. See rationale A.

D ) ethologically backward.

The issue of whether they are ethologically backward does not come up in the passage, since speculation centers on what human characteristics they will exhibit, characteristics that are either on par with human behavior or in advance of it.

Solution

Guess

99. Assume that the first extraterrestrials to contact Earth are a formerly warlike species that became pacifistic after suffering the consequences of a horrible war. This scenario most strongly supports the characterization of extraterrestrials favored by:

A ) James Trefil.

James Trefil suggests that evolution would have selected for aggressive aliens much like it selected for aggressive humans on Earthhe cautions that if evolution functions approximately the same way on other

B ) the author of the passage.

The passage author does not know if they would be pacific or warlike, which is why, for the author, the more important question to ask aliens would be about their philosophy rather than their technology

C ) the researchers mentioned in the second paragraph.

Page 28: AAMC MCAT Test 8 A

The researchers suggest that aliens might have experienced war sometime in their history since they would have acquired the wisdom to control war long before traveling in space so that by the time they contacted humans they would be no longer menacing.

D ) science-fiction writers.

The stereotypical aliens presented by science fiction writers are depicted in the passage as tending toward warlike, though in a comical way

100. The author implies that the reason red tides are difficult to control is that:

A ) phytoplankton can multiply rapidly, covering extensive areas.

The author describes how toxic algae blooms occur: Barring a shortage of nutrients or light, or heavy grazing by tiny zooplankton that consume the algae, the populations size can increase rapidly. Spread over large areas the phenomenon can be both visually spectacular and catastrophic.

B ) the presence of toxicity in seawater is likely to remain undetected.

The author points out that the science has matured in this area and toxins are therefore more readily discovered.

C ) the toxins increase to dangerous levels within the bodies of small fish.

The author indicates that this is not a control issue and may even be advantageous: From the human health standpoint, it is fortunate that herring, cod, salmon, and other commercial fish are sensitive to these toxins and, unlike shellfish, die before toxins reach dangerous levels in their flesh.

D ) human pollution of seawater is not adequately monitored.

Passage information suggests otherwise, that pollution monitoring has expanded: the global expansion in aquaculture means that more areas are monitored closely and that studies are showing a relationship between red tides and human-caused pollution.

Solution

Guess

101. Assume that a committee of environmentalists who are aware of the information in the passage is appointed to advise Congress on ways to reduce the problem of red tides. The members would probably recommend that:

A ) fisheries release their products only in areas that are free of algae.

This would be a less reasonable course of action because, even if the location of algae-free area could be determined, passage information indicates that could change, especially since algae can increase rapidly and algal cells are widespreadIn some cases a milliliter of seawater can contain tens or hundreds of thousands of algal cells.

B ) whales and other important marine life be driven away from affected areas.

This would not be a very practicable course of action since it would require vast resources and much effort to drive these animals out. There exists the additional possibility that more animals could enter the area once these were driven out and these efforts process would need to begin all over again.

C ) herbicides be used to destroy all toxic species of algae.

It is reasonable to assume that this measure could be actually counter-productive, since, much like a toxic red tide, the herbicides would cause toxic pollution that could harm marine wildlife and concentrate in the flesh of commercial fish, in turn, affecting human health if the herbicides did not kill the fish before reaching humans.

D ) plant nutrients be removed from wastewater before it is released into waterways.

Passage information suggests that removing nutrients would be the most reasonable course of action, given the data from pollution monitoring of coastal areas: Coastal waters receiving industrial, agricultural, and domestic waste, frequently rich in plant nutrients, should experience a general increase in algal growth. All phytoplankton species, toxic and nontoxic, benefit, but we notice the enrichment of toxic ones more.

Solution

102. If the authors information is correct, and if the trends mentioned continue, which of the following changes can be expected?

Page 29: AAMC MCAT Test 8 A

Guess

B ) The prevalence of large-scale die-offs of fish will increase.

Passage information indicates that red tides are increasing as revealed by long-term studies. As a result, more die-offs are likely: Algal toxins cause mortalities as they move though the marine food web. Commercial fish are especially sensitive to these toxins and die even before the toxins reach dangerous levels in their flesh, which indicates the likelihood of increased fish die-offs, especially with the increase in red tides.

C ) The relative number of algal species that are harmless will decrease.

According to the passage, a general increase in algae occurs, including nontoxic as well as toxic algae.

D ) The number of shellfish in coastal waters will decrease.

Unlike commercial fish, shellfish tend to concentrate toxins in their flesh rather than die.

Solution

Guess

103. Assume that a worldwide epidemic, especially severe among people in countries with a seacoast, is traced to poisoning by red-tide toxins. What is the most reasonable explanation of this phenomenon?

A ) Public anxiety in countries bordering oceans has increased their demand for medical attention. It logically

follows that an increase in anxiety could be attributable to a greater prevalence of red-tide poisoning, even if the number of people seeking medical attention grew disproportionately. In other words, the anxiety could be a natural outcome of the presence of an epidemic, but would not explain it.

B ) Contamination has spread to coastal areas in which people swim.

Passage information indicates that red-tide poisoning is caused by ingestion, not simply by swimming. The whales swimming in the infested waters had also been feeding. Toxins also concentrate in the livers and other organs of fish that are eaten by other animals, putting these animals at higher risk. The reason humans have been safe so far is because toxins do not concentrate to dangerous levels in the flesh of commercial fish before they die, so humans do not ingest the toxins by eating these fish.

C ) Food fish have developed increased resistance to the toxins.

This would be the most likely reason for the epidemic, since what protected human health before was the tendency of commercial fish to be especially sensitive to toxins and die off before the toxins could concentrate to dangerous levels in their flesh.

D ) Whales and other marine mammals are avoiding the toxic algae.

This is a nonsequiturit would not matter whether or not these animals were avoiding the algae if humans were not.

Solution

Guess

104. Which of the following findings would suggest that the authors concern about the danger of red tides is A ) The whales that died in 1987 succumbed to dramatic fluctuations in water temperature. B ) Chemical spills in the vicinity of the dead whales had caused an extreme level of contamination. C ) Red tides occur rarely, are easily identified, and grow slowly.D ) A readily available substance is an effective antidote to red-tide poisoning.

Page 30: AAMC MCAT Test 8 A

application of the antidote. 105. The primary purpose of the passage is to:

A ) identify the differences between two critical accounts of the growth of the novel in the U.S.

These differences are identified only in the opening paragraph and then refuted in the second paragraph before the author moves on to the main point, the prevalence of novels and readers of novels in the U.S. before the Civil War.

B ) trace the growth and influence of the magazine industry in the U.S. from 1840 to 1860.

This is not the main point but is a key point that supports the main idea by showing that fiction had a wide readership in the U.S. prior to the Civil War. See rationale C.

C ) question the idea that the social and intellectual atmosphere in nineteenth-century America was hostile to fiction.

The author first recounts the two stories literary historians tell about the novel in the U.S. prior to the Civil War, both of which cite a time period hostile to fiction. The author then goes on to refute this powerful critical myth by showing that a great number of novels were being read in America, especially during 1840 to 1860 when the number of periodicals expanded and published more than two thousand reviews of eight hundred separate novels, about half of them of American origin.

D ) offer contrary evidence to the argument that the Puritan tradition affected the growth of the romance in America.

The passage does not primarily offer contrary evidence to this particular idea alone, but, more generally, to the idea from which this idea concerning the development of the romance derives, namely, that an atmosphere hostile to fiction existed in the U.S. prior to the Civil War.

Solution

Guess

106. Assume that nineteenth-century American publishers could acquire European novels as easily as American novels, and publish them at a higher profit. Would this information, if true, further the passage authors argument?

A ) Yes; it would make more impressive the fact that one-half of the periodical reviews from 1840 to 1860 were

of American novels.

The author makes a key point about the expansion of the number of journals during this period and how these journals ran two thousand reviews of eight hundred novels, around half of them of U.S. origin, in order to show the great public interest in novels by U.S. authors. Therefore, it only follows that the number of American novels being read would be even more impressive, given that publishers in the U.S. could profit more by publishing European novels.

B ) Yes; it would reinforce the passage authors view that literary nationalism was not a viable force during this

period.

The author does not raise the issue of literary nationalism or hold this view. The statistic that half the novels were American only serves to show that many novels were being written in the U.S. as well as Europe and that the U.S. public was interested in reading them. It is not presented as evidence concerning literary nationalism or whether or not it was a viable force. See rationale A.

C ) No; it would merely demonstrate that American publishers made profit a top priority.

It would more logically follow in this case that publishers were as interested if not more interested in responding to the interest of the U.S. reading public as in making a profit. Otherwise, they would be more inclined to promote European novels over American. This only reinforces the authors point that the U.S. was a nation of novel readers rather than showing publishers were only interested in a profit.

D ) No; it would shed no light on the preferences of the American reading public for novels.

On the contrary, this would show that publishers were interested in responding to the publics interest in novels, especially American novels, since they could make more profit on European novels. See rationales A and C.

Solution

107. Which of the following assertions is NOT clearly supported by historical research provided by the passage author?

A ) Nineteenth-century American writers now considered major had difficulty supporting themselves by writing.

The author states: The small number of American fiction writers who are now called major did, evidently, have trouble supporting themselves as novelists. However, the author does not present the evidence and

B ) Many novels were being written and read in the U.S. from the 1840s through the 1860s.

The author supports this by citing the actual number of reviews of novels and the number of novels reviewed

Page 31: AAMC MCAT Test 8 A

Guess B ) Many novels were being written and read in the U.S. from the 1840s through the 1860s.

The author supports this by citing the actual number of reviews of novels and the number of novels reviewed during this period, two thousand and eight hundred respectively.

C ) American novels were well received by the major American periodicals of the mid-nineteenth century.

The author points out how, from perusing reviews during this period, American novels were well-received by reviewers to the point that staff members of magazines were concerned that reviewers were too quick to praise.

D ) The periodical press in the United States grew in size and influence during the 1840s.

The author provides ample statistics to support this point: There were fewer than 125 American magazines in 1825; by 1850 there were about 600. This included a number of journals that had over 100,000 subscribers and Horace Greeleys Tribune with over 200,000 subscribers.

Solution

Guess

108. What role does the sentence Scottish common sense philosophy should not be described as an American phenomenon play in the passage?

A ) It introduces a point that will be developed in detail later in the passage.

At no point later in the passage is this statement developed further. The statement is designed to counter one the critical myths referred to in the opening paragraph.

B ) It offers additional support for the points made in the preceding sentence.

The previous sentence in part reads: much of what Americans wrote and said about novels was derived from sources in the British Isles. In the context of the passage argument, the sentence in question is therefore a logical extension of this statement, elaborating on this point by means of specific reference to the origin of Scottish common sense philosophy in the U.S.

C ) It reminds the reader that hostility to the novel originated abroad.

The author never specifically makes the point that hostility to the novel originated abroad, only that expressed hostility to fiction was no less strong in England than in America, implying that this could have just as easily been a simultaneous development and that some Americans were simply influenced by what was said and wrote about regarding the novel in England.

D ) It suggests that Scottish common sense philosophy was hostile to fiction.

In citing one of the critical myths of literary history, the passage does not suggest that Scottish common sense philosophy itself was openly hostile to fiction, only that it joined with Puritanism to create an atmosphere hostile to fiction, a fine distinction. In addition, when considering its context within the passage, the sentence in question functions not to support this idea but to additionally support the preceding sentence. See rationale B.

Solution

Guess

109. Since the 1950s, some literary critics and historians have proclaimed the death of the conventional novel in America, attributing its demise to a widespread breakdown of social cohesiveness and an increase in individual isolation. This death-of-the-novel theory would be logically consistent with which aspect of the passage?

A ) The myth that the Puritan tradition made Americans hostile to the novel form

The author never explains specifically why the Puritan tradition was hostile to the novel Therefore there is

B ) The myth that the literary romance was compatible with the nineteenth-century American milieu

The breakdown of social cohesiveness and individual isolation are two reasons given for why the romance

C ) The research findings on nineteenth-century book reviews of American novels

The reviews far from proclaiming the death of the novel were largely enthusiastic giving tacit approval to

D ) The research findings on nineteenth-century attitudes toward the novel in England

Beyond implying that Scottish common sense philosophy may have influenced this hostility the author does

Page 32: AAMC MCAT Test 8 A

not explain the actual reasons behind this hostility, including specifically why Scottish common sense philosophy would contribute to this atmosphere. Therefore, there is no basis for comparing this to the death of the novel theory.

Solution

Guess

110. If the information in the passage is accurate, which of the following would one LEAST expect to find in a randomly selected American magazine published between 1840 and 1860?

A ) An article lamenting the abundant reviews of novels by nineteenth-century American novelists

This would be consistent with the passage argument, which pointed out that novels were being favorably reviewed and that some magazine staff members were concerned about indiscriminate puffery among reviewers.

B ) An editorial decrying Americans hostility to their own indigenous fiction

The author has pointed out that this hostility, one of the critical myths posited by literary historians, largely did not exist: a great many novels were written and published in America at this time of supposed hostility to fiction, and a great many more were being read. Therefore, such an editorial would be unlikely, especially since half the novels being read by the reading public in the U.S. were American.

C ) A favorable review of a new American novel copied from a British periodical

This would be consistent with the observation in the passage that much of what Americans wrote and said about novels was derived from sources written in the British Isles. Given the passage argument, this presumably could include favorable as well as unfavorable opinions about novels. Therefore, it would not be far-fetched to find a favorable review of an American novel copied from a British periodical in an American periodical, especially given a general atmosphere favorably inclined toward novels in which American novels were received warmly by reviewers.

D ) An essay reflecting the influence of Scottish common sense philosophy

Such an essay could very well appear, especially since the author points out that this philosophy had some influence upon life and thought in the U.S.. Another reason such an essay could very well appear would be if reviewers were aware of the influence of this philosophy on opinions concerning the novel and wanted to correct this opinion and offer guidance to readers, which, as the author points out, was one of the roles assumed by reviewers during this period.

Solution

Guess

112. Passage information suggests that the conventional novel was:

A ) a well-established literary form by the early nineteenth century.

Passage information suggests that the novel was not well-established: the conventional novel at that time was only in the process of becoming conventional.

B ) more common in England than in America.

Passage information does not state one way or the other whether the novel was more common in England than in the U.S. However, it was likely nearly as common in the U.S., given passage information that American periodicals ran two thousand reviews of eight hundred separate novels, half of which were American.

C ) similar in style and subject matter to the romance.

Passage information draws a distinct contrast between the social nature of the novel and the antisocial nature of the American romance.

D ) based on direct observation of social customs.

The author, in discussing one of the critical myths, draws attention to the view of some literary historians that the sparseness of American social life made conventional novels difficult, even impossible, to write. By extension, this would imply that the conventional novel was based on direct observation of social customs.

Solution

113. Suppose lists of best-selling books had existed during the period under study. Which of the following pieces of information derived from such lists would be useful in evaluating the accuracy of the passage authors

l i ?

Page 33: AAMC MCAT Test 8 A

Guess

III. The relative popularity of conventional novels and romances among American readers

A ) I only

Option III is also correct. See rationale D.

B ) III only

Option I is also correct. See rationale D.

C ) I and II only

The author makes no correlation between the length of the reviews the books received and their popularity, simply stating that reviews were of varying length with many of them highly favorable toward the novels they reviewed.

D ) I and III only

The author, even while acknowledging that reviewers and even groups of reviewers were expressing their own opinions in reviewing novels, nonetheless points out that, based on perusing reviews at the time, the reviewers had a fairly precise idea of public preferences and what guidance and correction to this public they were offering. The author also argues that novels were highly popular during this period and many were being written, thus dispelling the critical myth that would-be novelists were writing romances, which were more suited to conditions in the U.S. at the time. It follows, then, that these lists would need to reflect the relative popularity of romances and novels in order to evaluate the accuracy of the passage argument regarding the popularity of novels.

Solution

Guess

114. Wuthering Heights, written by Englishwoman Emily Brontë in 1847, is considered by most literary scholars to be a romance. What is the relation between this fact and the main argument of the passage?

A ) It supports the passage argument by showing that the romance was not a uniquely American form.

The main idea of the passage is to argue for the prevalence of the novels being written and read in the U.S. during this period and not to argue that the romance is a form unique to the U.S. Therefore, this observation about Wuthering Heights would not be relevant to the passage argument, much less support it. See rationale C.

B ) It strengthens the passage argument by demonstrating that literary scholars are often mistaken in their

conclusions.

The literary historians who argued that the romance was better suited to the U.S. during this period were not arguing that the romance was a form unique to the U.S. Therefore, it would not matter if an English novel was widely regarded to be a romance. Moreover, the main point of the passage is not to demonstrate that literary scholars are mistaken in their conclusions; therefore, this represents another reason why the observation regarding Wuthering Heights is irrelevant to the passage argument. See rationale C.

C ) It has little effect on the passage argument because the question of whether the romance is a distinctly

American form is not crucial to the authors point.

This would have little effect on the passage argument because the author focuses on the prevalence of novels being written and read during this period, and not on whether the romance was a distinctly American literary form in this period as some literary historians claim.

D ) It undermines the passage argument because it suggests that an atmosphere of alienation had spread to

England from America.

Since the passage argument is not about the relationship between alienation and romances, the point about Wuthering Heights as evidence of the spread of alienation to England, if true, is irrelevant to the argument. See rationale C.

115. Which of the following statements is inconsistent with information in the passage?

A ) Most people believe that the U.S. energy supply is nearly depleted.

The author notes that limits on the energy base in the United States began to surface in the late 1960s and early

B ) Environmentalism has caused fundamental changes in the U.S. energy policy.

Page 34: AAMC MCAT Test 8 A

The author discusses the major impact environmentalism had on the burning of coal, causing a switch to oil consumption by utility companies, which then tripled between 1968 and 1973.

C ) The use of domestic oil has been declining in the U.S. in recent years.

The author refers directly to the decline of U.S. oil production.

D ) Since the 1970s, the U.S. government has consistently supported the soft-energy approach.

The author states the opposite: An increasing number of individuals and communities in the U.S. are shifting to the soft path. A more rapid spread of this approach is being hindered by government (taxpayer) subsidies of the hard-path approach.

Solution

Guess

116. The author of the passage probably most strongly supports:

A ) the acquisition of solar technology companies by large corporations.

The author conveys concern about this because of how the energy companies began acquiring copper mines, since copper is a key part of solar heating and cooling systems. The author implies that this is part of this larger attempt to dominate the solar power industry and control energy production: Worried that every rooftop could become its own power plant and sensing that the cry for solar energy was a revolt against huge companies, utilities, and staggering electric bills, large corporations spent a share of their public relations budget playing down the solar messiahs. At the same time, they began buying up solar technology companies.

B ) increased utilization of low-sulfur oil by utility companies.

This is unlikely, since the author indicates support for solar energy. See rationale D.

C ) a switch to nuclear reactors as the primary energy source.

Since nuclear power is part of the hard-path approach, the author would likely not support this, since the author is already critical of subsidization of hard-path energy sectors. See rationale D.

D ) substantial federal subsidies for solar energy research.

The author indicates that taxpayer-supported government subsides of the hard-path approach are hindering the development of the soft-path approach. This includes inadequate access to capital for the development of solar energy resources. This criticism of subsidies for the hard-path approach and of inadequate capitalization of solar energy indicates that the author would likely support subsidies for solar power.

Solution

Guess

117. The author of the passage seems to hold the opinion that:

A ) U.S. utility companies are unresponsive to public environmental concerns.

The author describes how U.S. companies responded to environmental concerns by switching from coal to imported low-sulfur oil.

B ) the U.S. environmental movement has not fully confronted the energy problem.

The author brings up the environmental movement mainly to show the number of ways it has influenced public policy, including in the area of pollution control, which has influenced the switch from coal to low-sulfur oil. Beyond that, the author does not discuss how the environmental movement has or has not confronted the energy problem.

C ) large U.S. corporations act primarily according to the profit motive.

By discussing the attempts by energy companies to dominate solar power through acquisitions while at the same time disparaging solar power, the author is suggesting that large corporations are acting mainly through the profit motive in this attempt to remove an energy competitor that would allow individual buildings and homes to create their own power source rather than be dependent on the power the companies provide.

D ) the U.S. Congress is committed to reforming the countrys energy policy.

Nothing indicates that the author thinks the government is committed to reforming energy policy, only that the author is critical of the government for subsidizing hard-path approaches such as oil and gas.

Page 35: AAMC MCAT Test 8 A

Solution

Guess

118. Which of the following forms of legislation would an advocate of the soft-energy path probably support?

I. Tax credits for corporations that install solar panels in office buildings II. A mandate to increase the ratio of soft- to hard-energy sources by a specified amount within ten years

III. Cash incentives to homeowners who convert their heating systems from oil to natural gas

A ) I only

Option II is also correct. See rationale B.

B ) I and II only

The author is critical of the lack of capitalization for solar energy development and government subsidies of hard-path approaches to the energy problem such as oil and gas Therefore, it can be inferred from this that the author would likely advocate tax credits to install solar panels in office buildings and also support an effort to mandate an increase in the ratio of soft- to hard-energy sources with in ten years.

C ) I and III only

These cash incentives would represent a hard-energy approach which the author does not favor. See rationale B.

D ) II and III only

Option III is incorrect. See rationale C.

Solution

Guess

119. Which of the following inferences is justified by information in the passage?

A ) The U.S. leads the world in the use of solar energy.

Passage information suggests otherwise: In 1984about 18% of all primary energy used in the world and 8.5% of that used in the U.S. came from renewable solar energy resources.

B ) The burning of imported oil pollutes the air less than does the burning of coal.

The author points out: Concern about air pollution led to fuel switching, especially by electric utilities, away from domestically produced coal to low sulfur oil. This implies that low-sulfur oil is less polluting.

C ) Oil companies have a global monopoly on copper production. Passage information shows that oil companies

monopolize almost 60% of domestic copper production in the U.S., but does not indicate one way or the other that these companies similarly dominate world copper production.

D ) The consumption of natural gas has declined in the U.S. since the 1970s.

Passage information indicates that the hard-path approach proposes rapid expansion of oil and gas resources in addition to nuclear fission and coal. However, this proposed expansion is not the same as actual expansion and the author does not indicate anywhere in the passage whether natural gas consumption has actually increased or declined since the 1970s.

Solution

Guess

121. Suppose that an inventor has perfected a solar battery that would allow U.S. homeowners to convert rapidly and inexpensively from traditional electricity to solar power. The passage suggests which of the following scenarios as most probable?

A ) Utility companies attempt to purchase the patent on the battery.

The author describes how oil companies acquired copper mines and solar technology companies in an attempt to diversify into this new technology. Given this information, a similar attempt on the part of utility companies to purchase this battery is a most probable scenario.

B ) The government provides business grants to encourage mass production of the battery.

This is a far less probable scenario since the author criticizes the government for subsidizing hard-path approaches rather than soft-path technologies such as solar. It follows that the government would follow basically the same pattern by not offering business grants to encourage mass production of the battery.

C ) Hard-path energy corporations publicly praise the invention.

Page 36: AAMC MCAT Test 8 A

This is unlikely since the author points out that hard-path corporations disparaged solar power even as they were purchasing solar technology companies. It is probable that these corporations would respond the same way to this solar battery. As in the case of solar energy cited in the passage, the battery would similarly decentralize energy production into individual homes and buildings, which the companies would view as a threat.

D ) The nuclear power industry lobbies for reduced taxes for customers who do not convert to solar power.

The nuclear power industry, a hard-path industry, would likely not need to do this since, according to passage information, the government is already willing to subsidize hard-path approaches to energy.

Solution

Guess

122. Information in the passage suggests that which of the following changes would be most likely to accelerate U.S. progress along the soft-energy path?

A ) The price per barrel of imported oil rises from $25 to $50 within a few months.

Since a sharp rise in prices also implies a decrease in supply, this would accelerate a conversion toward soft-path energy technologies, which are nondepletable and emphasize diversification and dispersal of energy sources so as to avoid in the future the sort of dependence we now have on fossil fuels.

B ) An improved design for nuclear power plants makes radioactive leakage virtually impossible.

This would likely slow progress toward soft-path energy, since it would reduce concerns about nuclear power plant safety and the overall viability of this hard-path energy technology.

C ) Coal-burning plants install furnaces that eliminate the emission of air pollutants.

This would slow progress toward soft-path energy since the elimination of air pollutants would eliminate a major environmental concern about coal-fired plants that is not a concern with soft-path energy such as wind and solar.

D ) Domestic oil production increases to the point that the U.S. is no longer dependent on imported oil.

The author point out: The turning point [in the consumption of oil] came in 1970, when U.S. oil production reached its peak and then began to decline. Although the author does not say so directly, this was likely a factor in the shift toward soft-path energy technologies on the part of the increasing number of individuals and communities in the U.S.. It follows, then, that an end to dependence on imported oil would favor hard-path energy over soft-path.

123. The primary message of the passage is that one should:

A ) understand the profound importance of sports.

The author makes several statements that emphasize this point: The institutions of the state generate a civil religion; so do the institutions of sport. The author then cites the example of the ancient and modern Olympics and the Super Bowl, which to varying degrees honor the gods and the state and are constructed around high ceremonies, rituals, and symbols. The author later reinforces this point: Sportsare organized and dramatized in a religious way. The author elaborates on this theme in a way that underscores this profound importance of sports in public life: sports flow outward into action from a deep natural impulse that is radically religious: an impulse of freedom, respect for ritual limits, a zest for symbolic meaning, and a longing for perfection.

B ) appreciate the religious history of sports.

While the religious history of sports plays into the passage argument in the way the author discusses the ancient Olympics, this religious history does not represent the main point but only supports it. See rationale A.

C ) recognize the patriotic impulses in sports.

The patriotic impulses in sports to which the author refers do not represent the main point of the passage but only support it. These impulses are but one dimension in the larger significance that sports takes on in public life, as the author points out: Most men and women dont separate the sections of their mind. They honor their country, go to a place of worship, and also enjoy sports. All parts of their lives meld together.

D ) distinguish between the ceremony and the excitement of sports.

The author does not make this distinction but, instead, implies that the ceremony and the excitement and drama of sports represent one part of the whole: Sportsare organized and dramatized in a religious way.

124. A probable reason for the use of the word godward is the authors:

Page 37: AAMC MCAT Test 8 A

Solution

Guess

A ) faith in a divine presence that is apparent in daily life.

The author takes care to distinguish godward from the narrower sense of a divine presence, in particular, pointing out: I dont mean that participation in sports, as athlete or fan, makes one a believer in God, under whatever concept, image, experience, or drive to which one attaches the name.

B ) wish to use language that includes all persons, whatever their views on religion.

The author uses this word in the more generic sense of explaining the religious impulses in sports: Among the godward signs in contemporary life, sports may be the single most powerful manifestation. I dont mean that participation in sports, as athlete or fan, makes one a believer in God, under whatever concept, image, experience, or drive to which one attaches the name. Rather, sports drive one in some dark and generic sense godward.

C ) awareness of the similarity of popular spiritual movements to conventional religions.

Popular spiritual movements, or their relationship to conventional religions, are not even discussed in the passage. The author does discuss some of the ways sports takes on the qualities of conventional religion, for example, pointing out how the rituals, vestments, and tremor of anticipation involved in sports events are like those of religions.

D ) opinion that attendance at sporting events will lead to a religious revival.

The author never implies that interest in sports will lead to a religious revival, only that contemporary sports incorporate the qualities of religious observances. See rationale C.

Solution

Guess

125. The reference to the classical position of sports suggests that the author:

A ) respects the devoutness and patriotism of athletes.

The author does not suggest that athletes are necessarily devout or patriotic, even suggesting otherwise: The athlete may of course be pagan, but sports are, as it were, natural religions.

B ) admires the skill of the athletes of ancient Greece.

The author does not use the term classical position in reference to the skills of ancient Greek athletes, nor even allude to their skills anywhere in the passage. See rationale C.

C ) understands the historical function of athletics.

The author traces the contemporary phenomenon of sports as a kind of civil religion back to the ancient Greeks. Therefore, the author uses this terminology in the sense of recognizing this on-going function of sports in public life: The ancient Olympic games used to be both festivals in honor of the gods and festivals in honor of the stateand that has been the classical position of sports ever since (italics added)

D ) accepts the domination of athletics by religion and politics.

The author neither rejects nor accepts the domination of athletics by religion and politics by using this terminology. The author simply uses it more objectively in order to show how the relationship between sports, politics, and religion is a long-standing historical fact that extends into the present. See rationale C.

Solution

Guess

126. Which of the following findings would most weaken the authors assertions about the value of sports in the U.S. A ) People would rather attend religious ceremonies than sporting events.B ) Sports fans fail to recognize the religious impulse in their enthusiasm.C ) Only the conventionally religious experience the spiritual dimension of sports.

Page 38: AAMC MCAT Test 8 A

dimension of sports, the author would need to substantially revise this analysis.

D ) Players on a team have stronger feelings about the sport than do fans of the team.

This would not be relevant to the argument, since the feelings of athletes in relation to the feelings of fans in a sport is a matter separate from the matter of the religious dimension of sports.

Solution

Guess

127. The statement to lose is a kind of death is an example of the authors emphasis on the:

A ) extent of the emotional investment in sports.

The author makes this statement in the context of discussing the extent of the emotional investment in sports: In the language of Paul Tillich, sports are manifestations of concern, of will and intellect and passion. In fidelity to that concern, one submits oneself to great bodily dangers, even to the danger of death. Symbolically, too, to lose is a kind of death.

B ) importance of honoring losers as well as winners.

Honoring losers as well as winners does not enter into the passage discussion at all, much less in reference to this particular point.

C ) unacceptability of losing a sporting competition.

The author, in a sense, makes a case for the acceptability of losing here, by implying that losing, like death, is part of the order of things, a point made earlier in the passage when the author refers to how sports express a larger order of things that includes fate, destiny, and a sense of participation in the rhythms and tides of nature itself. The author implies that one who participates in sports submits oneself to losing the way one submits to great bodily dangers, even to the danger of death. In other words, one accepts that one may not only lose, but die trying.

D ) consequences of an exaggerated concern with sports.

While the inference could be made that the religious dimension of sports represents evidence of an exaggerated concern with sports, this inference would be external to the passage argument, since the author nowhere states or implies an exaggerated concern with sports. The author suggests otherwise, that concern for sports is not exaggerated but represents a deeply human experience: They feed a deep human hunger, place humans in touch with certain dimly perceived features of human life within this cosmos, and provide an experience of at least a pagan sense of godliness.

Solution

Guess

128. The passage suggests that behaviorism and type-type identity theory share which of the following weaknesses?

A ) They oversimplify mental states by reducing them to physiological states.

This is only true of type-type identity theory, which holds that every type of mental experience, such as pain, is identical to a corresponding type of physical event, such as the firing of C-fibers.

B ) They provide no reason for the existence of pain or other mental states.

Type-type identity theory explains pain as the firing of C-fibers. Behavioralism explains mental states as being created by external events, for example, the belief that it is going to rain consists of dispositions to act as if it was going to rain. The author criticizes this view as being limited to a belief taken in isolation and failing to provide a context: Whether you are disposed to take your umbrella with you depends not just on your belief that it will rain but also your desire not to get wet, your perception of the umbrella in the corner, your further belief that umbrellas are good for keeping rain off, and so on.

C ) They ignore the context provided by associated beliefs and feelings.

The passage indicates that behavioralism does not take into account other factors: You may have a pain in your toe without having a disposition to complain or move your toe because you may be a super Spartan who does not want to give any sign of what you are feeling. This statement reflects the authors view that behavior in relation to the toe is governed by its context, namely, the beliefs associated with not wanting to give the impression of being in pain. The type-type identity theory holds that mental experiences have corresponding physical events. The passage notes that this theory is controversial because of its implication that the physical basis for any type of mental experience must be the same in different creatures, which ignores the different contexts for the mental life of different creatures.

D ) They assume that all mental events are determined by external physical events.

This is only true of behavioralism.

129. In 1979, opioid peptides, which produce pain relief in humans, were found in earthworms. According to passage

Page 39: AAMC MCAT Test 8 A

Solution

Guess

information, someone who argues on the basis of this evidence that earthworms may be capable of feeling pain is most likely to be a:

A ) behaviorist.

A behavioralist would look for behavior in earthworms that acts in ways to remove themselves from the pain.

B ) type-type identity theorist.

The passage points out that one implication of type-type identity theory is that the physical basis for any type of mental experience must be the same in different creatures. Therefore, the assumption can be made that, if opioid peptides relieve pain in humans, then earthworms must feel pain in order to need the pain relief provided by opioid peptides.

C ) functionalist.

A functionalist would look for a greater context for why opioid peptides occur in earthworms, including their whole system of interdependent functions as earthworms, which suggests that opioid peptides may occur in earthworms for quite different reasons than they occur in humans

D ) nonscientist.

This assumes that a nonscientist will have the expertise to isolate opioid peptides in humans and earthworms, know how they function and why, and be interested in looking into the occurrence of these in humans and earthworms, none of which is likely.

Solution

Guess

130. The author apparently believes that crude behaviorism:

A ) does not take internal context sufficiently into account.

The author reflects concern that crude behavioralism does not take context into account: A particular belief taken by itself cannot be identified with particular behavioral dispositions because the relevant behavioral dispositions are determined by various beliefs taken together plus various desires and other mental states.

B ) should be replaced by type-type identity theory.

The author does not believe this, and notes that type-type identity theory has a similar problem to behavioralismit is too reductive in its outlook.

C ) can explain only actions that are predictable.

The author would not make this point and, arguably, would not believe any action is predictable, but is governed by context, which is variable. See rationale A.

D ) assumes that beliefs and feelings do not exist.

The author does not make this assumption. Earlier in the passage the author implicitly acknowledges that behavioralism assumes the existence of beliefs and feelings, since the author uses one example of behavioralism based on response to pain and another example of behavioralism based on response to a belief.

Solution

Guess

131. Suppose that a mountain climber is offered a replacement for the rope by which the climber is suspended. If the climber accepts the replacement rope, with which of the following conclusions about this acceptance would the

A ) It constitutes evidence of a belief in the weakness of the original rope.

B ) It is one of many possible results of a belief in the weakness of the original rope.

C ) It is probably influenced by beliefs other than a belief in the weakness of the original rope.

Page 40: AAMC MCAT Test 8 A

may not result in such obvious external outcomes, which according to critics, behavioralism fails to account for.

D ) It indicates the operation of brain processes associated with changing ropes while climbing.

The author would be more likely to agree with this conclusion, since it reflects a view that several associated brain processes are at work here. See rationale A.

Solution

Guess

132. If asked, most adults would affirm that giraffes do not wear hats in the wild. Such beliefs are apparently constructed from relevant general knowledge rather than stored as explicit information. Is the conclusion that certain beliefs are only implicit consistent with functionalism as the author explains it?

A ) Yes, because implicit beliefs come into existence when there is a need for them

The author explains: Functionalism holds that mental events of a certain sort are to be identified as those underlying events, whatever they are, that function in a relevant way. The affirmation that giraffes do not wear hats in the wild is an example of a mental event based on the underlying event of general knowledge functioning in a relevant way.

B ) Yes, because implicit beliefs are underlying physical events rather than dispositions to behave

Implicit beliefs are mental events, not physical events. Type-type identity theory, not functionalism, would hold that they are physical events.

C ) No, because implicit beliefs are irrelevant to desires, intentions, perceptions, and feelings

Functionalism sees beliefs as relevant: to understand the way beliefs function, you have to understand the way beliefs function in relation to desires, intentions, perceptions, emotions and inference.

D ) No, because implicit beliefs have no function

Functionalism holds that beliefs are one of the functional processes. See rationale C. 133. According to the authors description of the monitor theory, learned rules are most useful when a student of a second

language is:

A ) showing the habits of an underuser.

While underusers may have low levels of learned competence, learned rules are not useful to them, since underusers could still achieve very high levels of communicative accuracy through acquisition. The only situation in which learned rules would be useful to underusers is when they are taking a grammar testsee rationale C.

B ) beginning to study the language.

The author points out that learned rules are not very helpful in normal communicative situations, particularly in the beginning stages. Monitor theory holds that acquired rather than learned rules are more useful to second-language students.

C ) taking a grammar test.

In explaining the monitor theory, the author notes: Krashens monitor theory thus explains that oral proficiency in communication is not necessarily related to the ability to achieve high scores on standard grammar tests. Therefore, it follows that learned rules would be most useful when students are taking a grammar test.

D ) attempting to engage in conversation.

The author points out that most people cannot monitor learned rules in normal communication and that learned rules can actually interfere with oral communication.

Solution

Guess

134. According to the author, Krashen would classify people who frequently correct their own grammar while A ) superusers. B ) overusers.

Page 41: AAMC MCAT Test 8 A

effort on correctness that it often seriously interferes with communication.

C ) optimal users.

Optimal users find a proper balance by monitoring their speech only enough to improve their grammar but not so much that it interferes with their ability to communicate smoothly.

D ) underusers.

Underusers rarely use their learned competence [but] might still achieve very high levels of communicative accuracy through acquisition.

Solution

Guess

135. Which of the following findings about the value of grammar tests would be most troublesome for the monitor theory?

A ) The scores of beginning language students are good predictors of their oral proficiency.

Monitor theory asserts that there is little relationship between good performances on grammar tests and oral proficiency. Therefore, this finding would represent a strong challenge to monitor theory.

B ) The scores of beginning language students are poor predictors of their oral proficiency.

This would prove far less troublesome to monitor theory since this exception is explained by the fact that many language instructors tend to be superusers who are able to use knowledge of grammar rules to improve oral proficiency.

C ) The scores of language instructors are good predictors of their oral proficiency.

In explaining the monitor theory, the author notes: Krashens monitor theory thus explains that oral proficiency in communication is not necessarily related to the ability to achieve high scores on standard grammar tests. Therefore, it follows that learned rules would be most useful when students are taking a grammar test.

D ) The scores of language instructors are poor predictors of their oral proficiency.

This would still be consistent with monitor theory in the case of those language instructors who are not superusers. See rationales A and C.

Solution

Guess

136. The author states that underusers might achieve very high levels of communicative accuracy entirely through acquisition. Which of the following items of information provided elsewhere in the passage most helps to clarify this statement?

A ) Children develop their ability to speak in informal situations.

The author cites overwhelming evidence that students who wish to communicate must acquire this ability in much the same way that speakers, adults or children, acquire it in natural situations. Given this, underusers most resemble children in how they acquire oral proficiency.

B ) Learned rules of grammar have limited usefulness for language students.

This does not clarify why underusers achieve language proficiency, and is only relevant to the extent that learned rules have especially limited usefulness for underusers.

C ) Some people can apply learned rules so efficiently that their monitoring is not noticeable.

These people, namely, superusers are so different from underusers in how they acquire language proficiency that their example does not clarify why underusers achieve fluency.

D ) Modifications of utterances are usually initiated by prior knowledge.

Since this kind of modification can only happen through learned competence, which is low in underusers, this does not clarify how underusers acquire oral proficiency.

Solution

137. The passage suggests that an effective way for second-language learners to improve their fluency would be to:

A ) drill themselves repeatedly on everyday vocabulary.

This would only allow second-language students to increase their vocabulary, which monitor theory suggests would be of limited use, especially if they could not put this vocabulary to use in actual situations. See

B ) use audiotapes to learn grammar while sleeping.

Page 42: AAMC MCAT Test 8 A

Guess

B ) use audiotapes to learn grammar while sleeping.

Assuming that this method is effective, it would only be effective for learning grammar and would not be effective in normal communicative situations.

C ) use the second language to request information.

Using the second language to request information would be an example of the kind of natural situation that, according to monitor theory, allows language learners to improve their fluency.

D ) perform mental exercises to increase their memory.

This would only be effective for cognitive- based knowledge such as learning about correct grammar and syntax, which require memorization. Monitor theory would view this as having very limited use for improving fluency, which requires practice in natural situations, not memorization. See rationale B.

138 Essay 1 139 Essay 2 140. Which of the following conditions could produce rickets?

I. Metabolic deficiency of parathyroid hormone

II. Impairment of conversion of vitamin D to its active form

III. Inability of the active form of vitamin D to act on its target tissue

A ) I only

B ) I and II only

C ) I and III only

D ) II and III only

The passage states that rickets is caused by insufficient vitamin D activity. Insufficient vitamin D activity would reduce the ability of the body to absorb ingested calcium from the small intestine. To maintain calcium levels in the blood plasma, parathyroid hormone would promote the breakdown of bone tissue, causing the bones to become weak. If there were a metabolic deficiency of parathyroid hormone, the body would be unable to break down bone tissue (option I), causing a higher than normal ratio of mineral to organic matter in the bones instead of a lower than normal ratio. However, if the body were unable to convert vitamin D to its active form, or if vitamin D were unable to act on its target tissue, overall vitamin D activity would be impaired (options II and III), which can lead to rickets. Thus, D is the best answer.

Solution

Guess

141. Why do calcium supplements often include vitamin D?

A ) Vitamin D is needed to prevent rickets.

B ) The activated form of vitamin D stimulates the absorption of calcium into the blood.

Activated vitamin D acts on the small intestine to stimulate the absorption of calcium into the bloodstream. The inclusion of vitamin D in calcium supplements would ensure that vitamin D is present in the body to help promote this absorption. Thus, B is the best answer.

C ) The activated form of vitamin D enhances the action of calcitonin.

D ) The activated form of vitamin D enhances the uptake of calcium by bone tissue.

Solution

142. A low level of calcium in the plasma will trigger an increase of:

I. osteoclast activity. II. parathyroid hormone.

III. vitamin C.

A ) I only

Page 43: AAMC MCAT Test 8 A

Guess B ) I and II only

When the level of calcium in the blood plasma is low, the body responds by mobilizing stores of calcium from the bones via the activity of parathyroid hormone. Parathyroid hormone will increase the number of osteoclasts, which break down bone cells. Therefore, one would expect an increase in both parathyroid hormone and osteoclast activity in order to increase the level of calcium in the blood plasma (options I and II). However, vitamin C (option III) promotes bone formation, a process that would further lower the calcium level in the plasma. Thus, B is the best answer.

C ) I and III only

D ) II and III only

Solution

Guess

143. Under what condition would the level of calcitonin tend to increase?

A ) When there is a dietary deficiency of calcium

B ) When there is a dietary deficiency of vitamin D

C ) When the level of calcium in the plasma is high

Calcitonin reduces bone resorption. Bone resorption occurs when the level of calcium in the blood plasma is low, but resorption is not needed when the level of calcium is high. Therefore, resorption would be reduced by calcitonin under conditions in which the level of calcium in the plasma is high. Thus, C is the best answer.

D ) When the level of parathyroid hormone is too low

Solution

Guess

144. Which of the following persons would be most likely to have rickets?

A ) A child with a dietary deficiency in fat-soluble vitamins living in a tropical climate

B ) A child with a dietary deficiency in fat-soluble vitamins living in a northern climate

The passage states that vitamin D is nonpolar and that it can be obtained through the action of ultraviolet rays on the skin. Nonpolar molecules tend to be lipid-soluble rather than water-soluble, and exposure to ultraviolet rays tends to be less in northern climates than in tropical climates. Of the choices given, a child who has a dietary deficiency of fat-soluble vitamins and lives in a northern climate would be most likely to develop rickets. Thus, B is the best answer.

C ) A child with a dietary deficiency in water-soluble vitamins living in a tropical climate

D ) A child with a dietary deficiency in water-soluble vitamins living in a northern climate

Solution

Guess

145. What would be the result of complete removal of the parathyroid glands?

A ) Severe neural and muscular problems due to deficiency of calcium in the plasma

Removal of the parathyroid gland would lead to hypocalcemia, a condition of low blood calcium, resulting from the lack of parathyroid hormone. This would cause increased neuromuscular excitability because of the change in membrane potential, which under normal physiological conditions, is partially kept in balance with extracellular calcium. Typically, the person would eventually die from severe respiratory muscle spasms. Thus, A is the best answer.

B ) An increase in calcitonin production to compensate for calcium deficiency in the plasma

C ) A drastic change in the ratio of mineral to matrix tissue in bones

D ) Calcification of some organs due to accumulation of calcium in the plasma

146. Which of the following experiments would provide the best supporting evidence that neutrophils are the cause of the

reperfusion injury?

Page 44: AAMC MCAT Test 8 A

of tissue damage is reduced

The scientists were testing the hypothesis that toxins released by neutrophils are the cause of reperfusion injury. By repeating the experiment in neutrophil-depleted mice, they can assess whether the degree of tissue damage is reduced in the absence of neutrophils. A reduction in the amount of damage would implicate a role for neutrophils in the injury process. Thus, B is the best answer.

C ) Repeating the experiment with another antibody directed against the entire alpha/beta heterodimer, and examining

whether the degree of tissue damage is reduced

D ) Repeating the experiment with another antibody directed against the beta subunit, and examining whether the degree

of tissue damage is reduced

Solution

Guess

147. Information in the passage suggests most strongly that the function of the beta subunit involves:

A ) adhering neutrophils to the endothelium.

The passage states that the beta subunit is part of a heterodimer receptor present on the membrane of neutrophil cells that plays a role in the adhesion of neutrophils to endothelial cells. When antibodies were directed against the alpha subunit, no subsequent reduction in tissue damage occurred, implying that the neutrophils were still able to adhere to the endothelial cells. However, when antibodies were directed against the beta subunit of the receptor, a reduction in tissue damage was observed. This implies that when the beta subunit is blocked by antibodies, the receptor is no longer able to perform its function of helping the neutrophil adhere to endothelial cells. Thus, A is the best answer.

B ) transferring proteinases from endothelium to neutrophils.

C ) hydrogen bonding with the alpha subunit.

D ) the generation of antibody against the subunit.

Page 45: AAMC MCAT Test 8 A

Solution

Guess

148. The scientist applied antibody B to lab animals at different stages during ischemia and reperfusion. The animals were then studied for 10 days, and the results are shown in Figure 1.

Figure 1 Treatment of animals with antibody B at different time points during an ischemia/reperfusion experiment

The figure shows that the animals can be protected from tissue injury if the antibody treatment is received:

A ) at any time during ischemia or reperfusion.

B ) after reperfusion only.

C ) during reperfusion only.

D ) before reperfusion only.

The graph shows that when the antibody was administered during reperfusion the amount of tissue damage that occurred was nearly the same as that of the control, which had not received antibody treatment. However, both times that antibody treatment was administered prior to reperfusion, there was a substantial reduction in tissue damage as compared to that of the control. Thus, D is the best answer.

Solution

Guess

149. The scientist claimed that antibody B offers a better means for preventing organ injury than agents such as free radical or protease inhibitors. Which of the following reasons offers the best support for this claim?

A ) Antibody B is a high-affinity antibody; therefore, it will not be rejected by the patient.

B ) Antibody B can block the initiation of events that result in the release of harmful, biologically active

molecules.

Administration of antibody B reduced tissue damage following ischemia, presumably by preventing neutrophils from being able to adhere to the vascular endothelium. If this truly is the case, antibody B treatment would prevent neutrophils from invading the ischemic tissue, avoiding the release of the toxic chemicals such as free radicals and proteases. This should be more effective than trying to minimize damage from the toxins after they have already been released. Thus, B is the best answer.

C ) Antibody B is a very specific antibody; therefore, it will not recognize anything else other than the beta

subunit.

Page 46: AAMC MCAT Test 8 A

D ) Antibody B exhibits a high half-life and can be used at any dosage at any time.

Solution

Guess

150. The scientist wanted to use antibody B clinically (i.e., to treat humans), but this proposal was rejected. Which of the following is the most logical reason for the rejection?

A ) Because the antibody was generated in the mouse, it can never be used in humans.

B ) Because the antibody was generated in the mouse, repeated usage in the same patient would elicit the

production of human anti-mouse antibodies.

Antibody B was produced in mice; therefore, the human body would recognize it as a foreign antigen and mount an immune response, producing anti-mouse antibodies. Because an initial immune response to a specific antigen usually takes longer and is weaker than subsequent responses to the same antigen, a particular patient may not have a severe reaction to antibody B until after multiple exposures. Thus, B is the best answer.

C ) Because the antibody was generated in the mouse, it will not recognize human antigens.

D ) Because the antibody was generated in the mouse, it can only be used in vitro.

Solution

Guess

151. Which of the following situations would most likely occur in patients with defects in the neutrophil adhesion receptors during bacterial infections?

A ) A presence of pus at sites of bacterial infections

B ) An absence of pus at sites of bacterial infections

Pus is usually generated when neutrophils destroy cells by phagocytosis. To do so, neutrophils must first adhere to the cells that they are attacking. If the neutrophil has a defect in its adhesion receptors, it will not be able to bind to and engulf bacterial cells, and no pus will be generated. Thus, B is the best answer.

C ) An increase in the amount of circulating red blood cells

D ) A decrease in the amount of circulating white blood cells

Solution

Guess

152. A neutrophil has point mutations in the genes coding for the alpha and the beta subunits of the adhesion receptor. However, this cell can still migrate through endothelium. Which of the following conclusions about the effect of this mutation can be drawn?

A ) The cell cannot release toxic products such as prostaglandins.

B ) The cell has only functional beta subunits.

C ) The cell can bind to endothelium.

Cell adhesion is required for migration to occur. The neutrophils containing these particular point mutations can still migrate through the endothelium. This implies that the neutrophils can still bind to the endothelial cells. Thus, C is the best answer.

D ) The cell has a defective cell membrane.

153. Which of the following rationales explains which compound is the product of kinetic control?

A ) Its semicarbazone has the lower melting point.

B ) It forms faster at all temperatures because its rate of formation is independent of its path.

C ) Its reaction profile has the lower energy of activation.

The kinetically controlled product is formed faster than the thermodynamically controlled product because the energy of activation for the formation of the kinetically controlled product is lower. Thus, C is the best answer.

D ) Its reaction profile has the higher energy of activation.

154. Which semicarbazone is the product of thermodynamic control?

A ) Cyclohexanones, because the more stable product forms faster

B ) Cyclohexanones, because it contains more alkyl substituents to a double bond than does the other product

Page 47: AAMC MCAT Test 8 A

Solution

Guess

B ) Cyclohexanones, because it contains more alkyl substituents to a double bond than does the other product

C ) 2-Furaldehydes, because it is produced under equilibrium conditions and is more stable than the other

product

The melting point data in Table 1 indicate that the thermodynamically controlled product is the semicarbazone of 2-furaldehyde. The thermodynamically controlled product is the one that is formed under equilibrium conditions and is more stable. Thus, C is the best answer.

D ) 2-Furaldehydes, because its potential energy is higher

Solution

Guess

155. Based on its structure (shown below), the aldehydic proton in 2-furaldehyde should appear in its 1H NMR spectrum as a:

A ) singlet.

Because there are no hydrogens on the carbon next to the carbonyl carbon, the signal for the aldehydic proton is not split. The aldehydic proton will appear as a singlet. Thus, A is the best answer.

B ) doublet.

C ) triplet.

D ) quartet.

Solution

Guess

156. In each experiment, the crystals are collected on a Hirsch funnel and washed with two portions of cold water in order to:

A ) recrystallize the product.

B ) remove soluble impurities.

Rinsing the crystals with cold water will wash away the impurities soluble in cold water. Thus, B is the best answer.

C ) remove insoluble impurities.

D ) spread the crystals evenly over the surface of the funnel.

Solution

Guess

157. What is the hybridization of the carbonyl carbon in cyclohexanone?

A ) s

B ) sp

C ) sp2

The carbonyl carbon in cyclohexanone is bonded to three other atoms (two carbons in the ring and the carbonyl oxygen). There are three hybrid orbitals; the carbon is sp2 hybridized. Thus, C is the best answer.

D ) sp3

158. Assuming Hypothesis B to be correct, which of the following endocrine disorders would cause hypertension that could

NOT be rectified by physiologically normal kidneys?

A ) An excess of aldosterone

B ) An excess of glucagon

Page 48: AAMC MCAT Test 8 A

B ) An excess of glucagon

C ) A shortage of thyroxine

D ) A shortage of insulin

Solution

Guess

159. What mechanism probably would be responsible for the increased urine output induced by hypertension according to Hypothesis B?

A ) Increased blood flow to the bladder

B ) Increased renal tubular reabsorption of solutes and water

C ) Increased collecting duct permeability to water

D ) Increased glomerular filtration rate

The glomerular filtration rate is proportional to the glomerular capillary blood pressure minus the sum of the plasma osmotic pressure and the Bowmans capsule hydrostatic pressure. An increase to the systemic blood pressure would initially increase the glomerular capillary blood pressure, which would increase the glomerular filtration rate. Without a corresponding increase in the rate of tubular reabsorption of water, this would lead to an increase in urine output. Thus, D is the best answer.

Solution

Guess

160. If restriction of blood flow to the kidneys (by placing clamps on the renal arteries) resulted in an immediate but small increase in blood pressure, followed by the gradual development of severe hypertension, which hypothesis would these results best support?

A ) Hypothesis A, because the clamps increased the vascular resistance to blood flow

B ) Hypothesis A, because the clamps caused the kidneys to receive less blood

C ) Hypothesis B, because the kidneys were responding to decreased glomerular blood pressure

The reduced flow of blood through the renal arteries due to the clamps would cause a decrease in glomerular blood pressure. The kidneys respond to this drop in pressure by activating the reninangiotensin system of hormones. This increases the amount of sodium and water that is reabsorbed by the kidneys, therefore increasing blood volume and pressure. Thus, C is the best answer.

D ) Hypothesis B, because the volume of body fluids was probably decreasing

Solution

Guess

161. If blood pressure doubled and the resistance to blood flow increased by 50%, the amount of blood pumped by the heart would have:

A ) increased by 1/3.

The passage states that P = CO × VR. Solving the equation for cardiac output (CO) and letting the original

CO equal , the new CO would then equal . Reducing this equation shows that the new CO

is the old CO, or an increase of . Thus, A is the best answer.

B ) increased by 1/2.

C ) decreased by 1/3.

D ) decreased by 1/2.

Solution

162. According to Hypothesis A, enhanced activity of which of the following basic muscle types would be most likely to cause hypertension?

A ) Striated

B ) Smooth

C ) Cardiac

Page 49: AAMC MCAT Test 8 A

Guess C ) Cardiac

D ) Multinucleate

163. Synthesis of antibody proteins in eukaryotic cells is associated with what organelle?

A ) Nucleus

B ) Mitochondrion

C ) Endoplasmic reticulum

As secreted proteins, antibodies are translated by ribosomes attached to the rough endoplasmic reticulum. Thus, C is the best answer.

D ) Golgi apparatus

Solution

Guess

164. In the human "knee-jerk" reflex, the knee is struck and the lower leg jerks forward. Which of the following represents the complete pathway that the nerve impulse travels in effecting this response?

A ) Sensory neuron, motor neuron

The knee-jerk reflex is a simple monosynaptic stretch reflex. A tap to the tendon that connects the quadriceps to the patella activates a sensory neuron that directly synapses with a motor neuron in the spinal cord, causing the quadriceps to contract. Thus, A is the best answer.

B ) Sensory neuron, brain, motor neuron

C ) Sensory neuron, associative neuron, brain, associative neuron, motor neuron

D ) Sensory neuron, associative neuron, motor neuron, associative neuron, motor neuron

Solution

Guess

165. The most effective way to remove triethylamine during the workup of an organic reaction would be to extract the reaction mixture with aqueous:

A ) sodium bicarbonate.

B ) sodium bisulfite.

C ) sodium sulfate.

D ) hydrochloric acid.

Triethylamine would most likely be soluble in an organic solvent. Washing with aqueous hydrochloric acid will result in the formation of triethylamine hydrochloride. This salt is water-soluble and will be removed with the water wash. Thus, D is the best answer.

Solution

Guess

166.

Page 50: AAMC MCAT Test 8 A

What is the orientation of the tert-butyl and chloro substituents, respectively, in the predominant conformation of Compound 1?

A ) Axial, axial

B ) Axial, equatorial

C ) Equatorial, axial

Compound 1 is cis-1-tert-butyl-4-chlorocyclohexane. The predominant conformation of this compound is also the most stable one and has the cyclohexane ring in the chair conformation with the larger tert-butyl substituent in the equatorial orientation and the smaller chloro substituent in the axial orientation. Thus, C is the best answer.

D ) Equatorial, equatorial

167. Uric acid enters the urine both through filtration and secretion in the kidney. The process of filtration of uric acid in the

kidney takes place in the:

A ) glomerulus.

Filtrate is formed as fluid passes from the glomerular capillaries through the glomerular membrane into the Bowmans capsule. This region of the nephron is known as the glomerulus. Thus, A is the best answer.

B ) loop of Henle.

C ) distal convoluted tubule.

D ) proximal convoluted tubule.

Solution

Guess

168. Colchicine most likely relieves gout symptoms through what mechanism?

A ) Prevention of uric acid diffusion through cell membranes

B ) Inhibition of leukocyte phagocytosis of uric acid crystals

Phagocytosis requires that the cell change shape dramatically as it surrounds and engulfs large extracellular particles. Microtubules are one of the cytoskeletal elements that help determine cell shape. This function relies on the ability of the microtubules to disassemble and reorganize. The drug colchicine inhibits microtubule reorganization and would therefore inhibit phagocytosis of uric acid crystals by leukocytes. Thus, B is the best answer.

C ) Inhibition of uric acid crystal formation

D ) Maintenance of the pH optimum for PRPP synthetase

Solution

Guess

169. What nitrogenous base would promote the formation of uric acid crystals in gout?

A ) Cytosine

B ) Uracil

C ) Guanine

The passage states that uric acid is formed by the breakdown of purines to xanthine, a uric acid precursor. Guanine is one type of purine that is found in cells. Thus, C is the best answer.

D ) Thymine

Solution

170. In the patient described in the passage, the likely genetic basis of the increased levels of uric acid is a mutation:

A ) affecting an allosteric site of PRPP synthetase.

The passage indicates that the patient produces the normal amount of PRPP synthetase but its activity in

Page 51: AAMC MCAT Test 8 A

Guess

allosteric site on the enzyme has been affected. The mutant PRPP can most likely bind an intracellular molecule at an allosteric site, which changes the shape of the enzyme, enhancing its activity. This activity-enhancing molecule most likely is not present in the in vitro reaction mix. Thus, A is the best answer.

B ) affecting the active site of PRPP synthetase.

C ) in a promotor gene regulating the rate of transcription of the PRPP synthetase gene.

D ) in a gene coding for a transcription factor for the PRPP synthetase gene.

Solution

Guess

171. Some animals have developed the ability to excrete nitrogenous waste largely in the form of uric acid, which is nontoxic and does not require large amounts of water for its excretion. Considering its lifestyle, what animal would excrete nitrogen primarily in the form of uric acid?

A ) Wild pig

B ) Flying bird

Species of flying birds have evolved many characteristics that reduce their overall body mass, including the ability to excrete nitrogenous waste in the form of uric acid. Because the excretion of uric acid does not require large amounts of water, the amount of water that birds must ingest is decreased, making the birds lighter. Thus, B is the best answer.

C ) Carnivorous shark

D ) Herbivorous bony fish

172. The first step of the Hofmann rearrangement involves the abstraction of one of the protons on the nitrogen. The amide N

H proton is slightly acidic because the:

A ) resulting anion is resonance-stabilized.

The removal of the amide proton results in the formation of an anion with the negative charge on the nitrogen. The adjacent carbonyl group stabilizes the anion through resonance. A resonance structure of the anion can be drawn in which the negative charge resides on the oxygen. Thus, A is the best answer.

B ) NH bond is polar.

C ) aromatic ring is electron-donating.

D ) amide is not basic.

Solution

Guess

173. What is the major product when 2-phenylacetamide, below, is treated with bromine and aqueous base under the conditions in the passage?

A )

B )

C )

According to the passage a Hoffman rearrangement of a primary amide results in the loss of the carbonyl

D )

Page 52: AAMC MCAT Test 8 A

D )

Solution

Guess

174. It has been reported that under some conditions hydrolysis of the amide can compete with the Hofmann rearrangement. What product would be expected if this side reaction were important for Compound 1?

A ) Benzamide

B ) 3-Nitroaniline

C ) 3-Nitrobenzamide

D ) 3-Nitrobenzoic acid

Hydrolysis of Compound 1 would result in the formation of the corresponding carboxylic acid, 3-nitrobenzoic acid. Thus, D is the best answer.

Solution

Guess

175. The students monitored the conversion of Compound 1 to m-nitroaniline by infrared spectroscopy. The disappearance of which band would indicate that the starting material had been consumed?

A ) 1550 cm1

B ) 1650 cm1

As Compound 1 is consumed, the IR spectrum will show the loss of absorptions attributed to the carbonyl group. The C=O stretch of an amide appears at approximately 1650 cm1. Thus, B is the best answer.

C ) 2200 cm1

D ) 3300 cm1

Solution

Guess

176. The conversion of Compound 1 to m-nitroaniline can also be monitored by 13C NMR spectroscopy. The disappearance of the signal at which frequency accompanies the consumption of the starting material?

A ) 65 ppm

B ) 107 ppm

C ) 120 ppm

D ) 165 ppm

As Compound 1 is consumed, the 13C NMR spectrum will show the loss of the signal attributed to the carbonyl group. The carbon of the carbonyl group appears at approximately 160170 ppm. Thus, D is the best answer.

177. When glycerol reacts with three different fatty acids, how many stereogenic centers does the product triacylglycerol

contain?

A ) 0

B ) 1

When glycerol reacts with three different fatty acids, only carbon 2 in the resulting triacylglycerol is attached to four different groups. Thus, B is the best answer.

C ) 2

D ) 3

178. Which of the following formulas represents a general structure of a fatty acid salt produced in Reaction 1? (Note: Rn = R1 R2 or R3 )

A ) RnCH2

Na+

B ) RnCH2O Na+

Page 53: AAMC MCAT Test 8 A

Solution

Guess

B ) RnCH2O Na+

C ) RnC(O) Na+

D ) RnCO2

Na+

A fatty acid would have the general formula RnCO2H. The corresponding fatty acid salt would have the general formula RnCO2Na+. Thus, D is the best answer.

Solution

Guess

179. Which of the following is the most plausible explanation for the fact that the saponification of the triacylglycerol in the passage resulted in four different fatty acid salts?

A ) The triacylglycerol molecule consisted of four different fatty acid units.

B ) Glycerol was transformed into a fatty acid salt under the reaction conditions.

C ) One of the fatty acid salts was unsaturated, and it completely isomerized under the reaction conditions.

D ) One of the fatty acid salts was unsaturated, and a small percentage isomerized under the reaction conditions.

If one of the R groups in the triacylglycerol contained a carboncarbon double bond and if isomerization of the double bond occurred during the saponification reaction, four fatty acids would be obtained instead of three. Thus, D is the best answer.

Solution

Guess

180. A triacylglycerol can also be accurately described as a:

A ) triacid of glycerol.

B ) triether of glycerol.

C ) triester of glycerol.

The three acyl groups are joined to the glycerol backbone through ester linkages. Thus, C is the best answer.

D ) trihydroxy glycerol.

Solution

Guess

181. How much sodium hydroxide is needed to completely saponify a triacylglycerol?

A ) A catalytic amount, because OH is continuously being regenerated during saponification

B ) One-third of an equivalent, because each OH ion reacts to form three fatty acid salts

C ) One equivalent, because each OH ion reacts to produce one molecule of glycerol

D ) Three equivalents, because one OH ion is required to saponify each of the three fatty acid groups

One hydroxide ion is required to hydrolyze one ester linkage of a triacylglycerol molecule. Because there are three ester linkages in a triacylglycerol, three equivalents of sodium hydroxide will be needed to completely saponify the triacylglycerol. Thus, D is the best answer.

Solution

Guess

182. Which of the following statements most accurately describes the solubility properties of fatty acid salts?

A ) They are soluble in polar media only.

B ) They are soluble in nonpolar media only.

C ) They can partially dissolve in both polar and nonpolar media.

A fatty acid salt contains a long hydrocarbon chain, which is soluble in nonpolar solvents. The salt also contains the charged group CO2

Na+, which is soluble in polar solvents. Thus, C is the best answer.

D ) They are completely insoluble in both polar and nonpolar media.

183. A male taking excess testosterone may become infertile because of reduced spermatogenesis. According to Figure 2, this

could result directly from:

A ) an increase in inhibin concentration.

B ) a reduction in inhibin concentration.

Page 54: AAMC MCAT Test 8 A

B ) a reduction in inhibin concentration.

C ) a reduction in FSH concentration.

Figure 2 indicates that testosterone is part of a negative feedback loop that acts on the hypothalamus to prevent the release of GnRF. In the presence of testosterone, less GnRF would be present to stimulate the release of FSH from the pituitary gland, causing a decrease in FSH available to act on the Sertoli cells. Therefore, FSH is less able to promote and maintain spermatogenesis. Thus, C is the best answer.

D ) a reduction in LH concentration.

Solution

Guess

184. The cell type in the male reproductive system that is most analogous to the female ovum is the:

A ) spermatogonium.

B ) primary spermatocyte.

C ) spermatid.

D ) spermatozoon.

The mature ovum is the female gamete that has completed meiosis and contains the haploid number of maternally derived chromosomes. This makes it most analogous to spermatozoa, the mature male gametes that contain the haploid number of paternally derived chromosomes. Thus, D is the best answer.

Solution

Guess

185. Some drugs used in cancer chemotherapy kill proliferating cancer cells by selectively inhibiting various stages of the life cycle. Which of the following normal reproductive processes is likely to be most affected by the use of chemotherapy?

A ) Sertoli cell function

B ) Testosterone production

C ) Spermatogenesis

Spermatogenesis is the production of spermatozoa by the meiotic division of spermatocytes. Therefore, it would be the most affected by the use of chemotherapy. Thus, C is the best answer.

D ) Inhibin production

Solution

Guess

186. Which of the following hormones is(are) directly required for spermatogenesis?

I. Luteinizing hormone (LH) II. Follicle-stimulating hormone (FSH)

III. Inhibin IV. Testosterone

A ) IV only

B ) I and IV only

C ) II and IV only

Sertoli cells support and nourish the spermatocytes and promote the process of spermatogenesis. Spermatogenesis would not occur without Sertoli cells. The two hormones that directly stimulate Sertoli cells are FSH and testosterone. Thus, C is the best answer.

D ) I, II, and III only

Solution

187. On the basis of their function as nurse cells, which of the following organelles are most likely to be prominent in Sertoli cells?

A ) Golgi apparatus

B ) Lysosomes

Page 55: AAMC MCAT Test 8 A

Guess

B ) Lysosomes

C ) Mitochondria

D ) Cilia

Solution

Guess

188. Which of the following statements correctly describes the distinction between the exocrine and endocrine portions of the testis?

A ) The exocrine portion secretes only peptides; the endocrine portion secretes only steroids.

B ) The exocrine portion releases its products into ducts; the endocrine portion releases its products into the

blood.

Exocrine glands secrete their products through ducts; endocrine glands release their products into the bloodstream. Thus, B is the best answer.

C ) The exocrine portion secretes only cellular elements; the endocrine portion secretes only chemical

substances.

D ) The exocrine portion is the target tissue for the products of the endocrine portion.

189. The pancreas produces which of the following substances for the digestive system?

A ) Bile salts

B ) Emulsifier

C ) Gastric juices

D ) Proteolytic enzymes

The pancreas produces several proteolytic enzymes, which are released into the small intestine where they are converted to their active forms of trypsin, chymotrypsin, and carboxypeptidase. Thus, D is the best answer.

Solution

Guess

190. Which of the following characteristics clearly marks fungi as eukaryotes?

A ) They have cell walls.

B ) They contain ribosomes.

C ) They contain mitochondria.

One characteristic that distinguishes eukaryotic cells from prokaryotic cells is that eukaryotic cells contain membrane-bound organelles such as mitochondria. Thus, C is the best answer.

D ) They exhibit sexual reproduction.

Solution

Guess

191. If the ester shown below were hydrolyzed in acidic H218O, which product would be expected to contain 18O?

A ) CH3CO2H

The first step of the hydrolysis would be the protonation of the carbonyl oxygen. The H218O would then act

as a nucleophile, attacking the protonated carbonyl carbon. Cyclopentanol is the leaving group. The products are cyclopentanol and 18O-labeled acetic acid. Thus, A is the best answer.

B ) CH3OH

C )

D )

Page 56: AAMC MCAT Test 8 A

Solution

Guess

192. From which germ layer(s) do the tissues of the heart and blood vessels differentiate?

I. Ectoderm II. Mesoderm

III. Endoderm

A ) II only

The heart and blood vessels both differentiate from the mesoderm. Thus, A is the best answer.

B ) III only

C ) I and II only

D ) I and III only

Solution

Guess

193. The most effective method for producing an increase in the total amount of water lost through the skin during a certain period would be:

A ) inhibiting kidney function.

B ) decreasing salt consumption.

C ) increasing water consumption.

D ) raising the environmental temperature.

Water is lost through the skin primarily as a means to keep the body at normal temperatures. Therefore, raising the environmental temperature would cause a person to perspire, releasing water to the environment where it can evaporate and cool down the body. Thus, D is the best answer.

194. To determine the significance of UV reflectance by the dewlap, it would be most useful to compare the behavior of:

A ) sighted and sightless lizards, in response to flashing of the dewlap.

B ) lizards responding to flashing of normal dewlaps versus treated dewlaps that absorb UV.

To determine the significance of UV reflectance by the dewlap, it would be most useful to compare the response of lizards to dewlaps that reflect UV light to the response of lizards to dewlaps that do not reflect UV light. If the reflectivity of the dewlap is kept as the only variable, it can be determined whether a correlation exists between the lizards behavior and the reflectivity of the dewlap with which the lizards were flashed. Thus, B is the best answer.

C ) the five lizard species, when they are placed together in the same habitat.

D ) the five lizard species under illumination by red light only.

Solution

Guess

195. If these lizards use UV light in communication, a mutation that eliminated UV photoreceptors would probably cause the LEAST disadvantage to:

A ) species A.

B ) species B.

C ) species D.

D ) species E.

Assuming that the lizards use the UV-reflectivity of the dewlap primarily as a means of intraspecies communication, species E would most likely be least affected by a mutation that eliminated UV photoreceptors. Its dewlaps are the least UV-reflective of the five lizard species, which indicates that species E is least likely to rely heavily upon this form of communication in the first place. Thus, D is the best answer.

196. If Anolis lizards have X-Y chromosomal sex determination, the locus of a gene for the UV reflectance pigment:

A ) must be on the X chromosome.

Page 57: AAMC MCAT Test 8 A

Solution

Guess

B ) must be on the Y chromosome.

C ) must be on an autosome.

D ) could be on a sex chromosome or on an autosome.

Based on the information presented, the gene encoding UV-reflectance pigment could be on a sex chromosome or an autosome. The fact that the pigment is expressed in the dewlap, a structure found only in males, is not sufficient to eliminate any chromosome as the location of this gene. Thus, D is the best answer.

Solution

Guess

197. Two neighboring lizard populations would be considered separate species if:

A ) one population inhabited the forest and the other lived in a field.

B ) one population had a UV-reflective dewlap and the other did not.

C ) they did not communicate with each other.

D ) they did not interbreed and produce fertile offspring.

One of the key factors that determines a species is the ability to successfully breed and produce fertile offspring. Two organisms that do not meet this criteria are considered separate species. Thus, D is the best answer.

Solution

Guess

198. Which of the following conclusions about dewlap reflectance is supported by information in the passage?

A ) Lizard habitat is determined by dewlap reflectance for each species.

B ) High dewlap reflectance is most important in brightly lit habitats.

Figure 1 shows that the three species of lizards that live in unshaded fields possess dewlaps that are significantly more capable of reflecting UV light than do the two species of lizards that live in the shaded understory. This supports the conclusion that high dewlap reflectance is most important in brightly lit habitats. Thus, B is the best answer.

C ) High dewlap reflectance is most important in dimly lit habitats.

D ) Dewlap reflectance is highest at the blue end of the visible spectrum.

Solution

Guess

199. Dewlaps that reflect UV light would evolve by natural selection only if:

A ) individuals with UV-reflective dewlaps produced more offspring than did individuals without them.

Although many different types of adaptations may help an individual organism survive, they will not be passed on to the next generation unless the organism produces offspring, passing on the genes that cause the advantageous phenotype. To evolve by natural selection and become a general characteristic of the species, the genes that cause dewlaps to reflect UV light must become a significant portion of the gene pool, which will most likely occur if individuals with UV-reflective dewlaps produce more offspring than do individuals without them. Thus, A is the best answer.

B ) individuals with UV-reflective dewlaps were better able to communicate than individuals without them.

C ) individuals with UV-reflective dewlaps were less subject to predation than individuals without them.

D ) individuals with UV-reflective dewlaps mated more frequently than did individuals without them.

200. At what concentration of free actin will the + end of the microfilament grow faster than the end?

Page 58: AAMC MCAT Test 8 A

C ) At any concentration greater than 1 µM

Figure 1 shows that at free actin concentrations greater than 1µM, actin is added to the + end of a microfilament. At concentrations lower than 1µM, the + end loses actin subunits. The end of the microfilament does not begin to add actin until the free actin concentration is greater than 5.5µM. The rate at which actin is added to the + end is greater than the rate it is added to the end, implying that at any concentration greater than 1µM, the + end of the microfilament grows faster than the end. Thus, C is the best answer.

D ) At any concentration

Solution

Guess

201. Below is a diagram of a muscle sarcomere. Based on the passage, which statement best explains why the microfilament lengths do NOT change when the sarcomere shortens in a muscle contraction?

A ) The ends of the microfilaments are capped by Z lines, and the actin subunit concentration is kept above 1

µM in muscle cells.

B ) The ends of the microfilaments are capped by Z lines, and the + ends are capped by another protein.

Within a sarcomere, the microfilament length remains stable. Because one end of the microfilament is anchored in the Z line, actin monomers are prevented from being added to or subtracted from that end. This rules out the possibility of treadmilling. Therefore, to retain a stable length, both ends of the microfilament must be capped. Thus, B is the best answer.

C ) The actin subunit concentration is kept above 4 µM in muscle cells.

D ) The ends polymerize and the + ends depolymerize at the same rate.

Solution

Guess

202. The theory of force generation proposed in the passage is best supported by which of the following observations about Amoeba locomotion?

A ) Amoeboid movement stops upon exposure to cytochalasins.

The passage proposes that force is generated as a microfilament elongates and pushes against a structure such as the plasma membrane. This is representative of how an amoeba moves. Cytochalasins are drugs that inhibit the growth of microfilaments. Therefore, if amoeboid movement stops upon exposure to cytochalasins, microfilaments and their ability to elongate are both implicated as being necessary to generate the force for movement in an amoeba. Thus, A is the best answer.

B ) Amoeboid movement cannot occur if mitosis is blocked.

C ) Moving Amoeba cells produce more troponin than do stationary ones.

D ) The rate of movement is inversely proportional to the viscosity of the medium in which the Amoeba moves.

Solution

203. Based on Figure 1, at what free actin subunit concentration (or range of concentrations) will the microfilament treadmill?

A ) 0.25 µM

B ) 1.0 µM

Page 59: AAMC MCAT Test 8 A

Guess

Figure 1 shows that at a free actin concentration of 1.5µM, the rate at which actin subunits are added to the + end of the microfilament is equal to the rate at which actin filaments are removed from the end. This fits the definition of treadmilling. Thus, C is the best answer.

D ) Any concentration between 1.0 µM and 4.0 µM

Solution

Guess

204. Based on Figure 1, at what free actin subunit concentration (or range of concentrations) will both the + and ends of the microfilament experience a net loss of subunits?

A ) At any concentration below 1 µM

Figure 1 shows that at free actin concentrations less than 1µM, both the + and the ends of the microfilament experience a net loss of actin subunits. Thus, A is the best answer.

B ) Exactly at 1 µM

C ) At any concentration above 1 µM

D ) Only between 1 µM and 4 µM

Solution

Guess

205. Which of the following observations supports the hypothesis that microfilaments are involved in the release of viral particles?

A ) Exocytosis of viral particles from an infected cell is proportional to the rate of microfilament polymerization.

During exocytosis of viral particles the plasma membrane goes through many distortions of its shape. This requires polymerization of microfilaments, making it likely that the rate at which the viral particles can be expelled from the cell is dependent upon the rate of microtubule polymerization. Thus, A is the best answer.

B ) Treatment with phalloidin does not prevent the exocytosis of virus particles from the infected cell.

C ) No known virus carries genes coding for actin subunits.

D ) Some viruses have capsules composed of myosin.

Solution

Guess

206. Assuming that Amoeba uses microfilament-generated forces for locomotion, which of the Amoebas pictured below will move from left to right?

A )

B )

Page 60: AAMC MCAT Test 8 A

D )

Diagram D shows a cell with a free actin concentration of 2µM with the + end of the microfilament oriented toward the right side of the cell. At this concentration, the microfilament should show a net increase in length toward the right. When it reaches the plasma membrane it should continue to lengthen and generate a force that pushes the membrane outward, causing the cell to move in a left-to-right direction. Thus, D is the best answer.

207. In which of the experiments is a rearrangement of the carbon skeleton observed?

A ) 1 only

In Equation 1, one of the methyl groups moves to the adjacent carbon. This rearrangement is not seen in equations 2 and 3. Thus, A is the best answer.

B ) 2 only

C ) 3 only

D ) 2 and 3 only

Solution

Guess

208. Which set of reagents or condition could be used to prepare the alcohol in the following reaction?

A ) H2SO4/H2O

B ) Hg(OAc)2/THF-H2O; NaBH4/OH

C ) THF:BH3; H2O2/OH

Among the methods represented by the three equations in the passage, the one represented by Equation 3 is the only method of alcohol formation that results in the hydroxyl group adding to the less substituted carbon. Thus, C is the best answer.

D ) Heat

Solution

Guess

209. Alcohols have higher boiling points than hydrocarbons of comparable molecular weight. This is a result of:

A ) hydrogen bonding.

Alcohols contain hydroxyl groups, which enable them to undergo hydrogen bonding. Alcohols have higher boiling points because hydrogen bonds are strong intermolecular forces and more energy is needed to overcome them. Thus, A is the best answer.

B ) van der Waals forces.

C ) covalent bonding.

D ) resonance.

Page 61: AAMC MCAT Test 8 A

Solution

Guess

210. Another way to prepare an alcohol is via a Grignard synthesis. Which of the following reactants can be used in a Grignard reaction to produce the same alcohol that was produced in Experiment 1?

A )

2,3-Dimethyl-2-butanol was prepared in Experiment 1. As shown in answer choice A, the reaction of isopropylmagnesium bromide (a Grignard reagent) with acetone will result in the addition of the isopropyl group to the carbonyl carbon and the formation of 2,3-dimethyl-2-butanol. Thus, A is the best answer.

B )

C )

D )

211. The discovery that the amount of thymine equals that of adenine and the amount of guanine equals that of cytosine in a

given cell provides supporting evidence that:

A ) the Watson and Crick model of DNA is correct.

In the Watson and Crick model of DNA structure, the nitrogenous bases form hydrogen bonds with each other in a 1:1 ratio: guanine pairs with cytosine, and adenine pairs with thymine. This implies that the amount of guanine and cytosine would be the same, and the amount of adenine and thymine would be the same. Thus, A is the best answer.

B ) DNA is the genetic material.

C ) the genetic code is universal.

D ) the code for one amino acid must be a triplet of bases.

Solution

Guess

212. Consider an organism that has three pairs of chromosomes, AaBbCc, in its diploid cells. How many genotypically different kinds of haploid cells can it produce?

A ) 4

B ) 8

The number of different possible gametes that can be formed by diploid organisms as a result of independent assortment of chromosomes during meiosis can be calculated using the formula 2n where n is the haploid number of chromosomes. In this case, the haploid number is 3, making the number of different haploid cells 23, or 8. Thus, B is the best answer.

C ) 16

D ) 32

Solution

Guess

213. What is the net volume of fresh air that enters the alveoli each minute, assuming that the breathing rate is 10 breaths/min, the tidal volume is 800 mL/breath, and the nonalveolar respiratory system volume (dead space) is 150 mL?

A ) 65 mL

B ) 95 mL

C ) 6500 mL

The amount of air entering the lungs in a single breath, or tidal volume, is given as 800 mL/breath. Of that

Page 62: AAMC MCAT Test 8 A

800 mL only 650 mL reaches the alveoli per breath (800 mL of air inhaled minus 150 mL of nonalveolar respiratory volume). Therefore the net volume of air that reaches the alveoli each minute is equal to 650 mL/breath multiplied by 10 breaths/min, or 6500 mL. Thus, C is the best answer.

D ) 7850 mL

Solution

Guess

214. In mammals, which of the following events occurs during mitosis but does NOT occur during meiosis I?

A ) Synapsis

B ) The splitting of centromeres

One of the key differences between mitosis and meiosis occurs during their respective anaphases. During anaphase of mitosis, sister chromatids are pulled apart at the centromeres, each becoming an independent chromosome in the two diploid daughter cells. During anaphase I of meiosis I, homologous pairs of chromosomes are separated into the two daughter cells. However, each chromosome still consists of two sister chromatids joined to each other at the centromere. It is not until anaphase II of meiosis II that the centromere is split and the sister chromatids separate. Thus, B is the best answer.

C ) The pairing of homologous chromosomes

D ) The breaking down of the nuclear membrane

Solution

Guess

215. In a particular species of plant, tall vine depends on a dominant gene (T), and a pink flower is the result of the heterozygous condition of the genes for red and white flowers (Rr). What fraction of the offspring from the cross of a tall, pink plant (heterozygous for height) with a short, pink plant would be expected to be pink AND tall?

A ) 3/4

B ) 1/2

C ) 3/8

D ) 1/4

Based on the given information, the tall pink plant would have the genotype TtRr and could form the following gametes: TR, Tr, tR, and tr. The short pink plant would have the genotype ttRr and could form the following gametes: tR and tr. The possible genotypes of the offspring are displayed in the following Punnett square.

TR Tr tR tr tR TtRR TtRr ttRR ttRr tr TrRr Ttrr ttRr ttrr

Of these, 1/4 would be both pink (Rr) and tall (TT or Tt). Thus, D is the best answer.

Solution

Guess

216. During prokaryotic protein synthesis, translation begins as soon as the newly synthesized mRNA strand begins to extend from the DNA strand. This situation differs from that in eukaryotes, because eukaryotes:

A ) carry out translation without using ribosomes.

B ) transcribe mRNA molecules without using DNA.

C ) destroy most mRNA as soon as it is synthesized.

D ) localize the processes of transcription and translation in the nucleus and cytoplasm, respectively.

Eukaryotes have membrane-bound organelles including the nucleus, which contains the DNA. Transcription of DNA into RNA occurs in the nucleus. The RNA is then transported to the cytoplasm where ribosomes translate it into proteins. Thus, D is the best answer.